#13 Rosh Review

Pataasin ang iyong marka sa homework at exams ngayon gamit ang Quizwiz!

Question: What arterial blood gas finding is concerning for impending respiratory failure in an asthmatic?

Answer: A normal, elevated or rising carbon dioxide. Rapid Review Asthma Asthma: airway inflammation + bronchial hyperresponsiveness + reversible airflow obstruction PEF <50%: severe exacerbation Treatments: O2: maintain SpO2 > 88% ß-agonists: ↑ cAMP → bronchodilation Anticholinergics: ↓ bronchoconstriction Corticosteroids: ↓ inflammation, administer early Mg: severe exacerbations Non-invasive ventilation: ↓ work of breathing Mechanical ventilation: Objective: maximize expiratory time Low respiratory rate High inspiratory flow rate Maintain plateau pressure <30 cm H2O Permissive hypercapnia to avoid breath stacking

Question: What are Roth spots and Osler nodes?

Answer: All are dermatologic manifestations of left-sided endocarditis. Roth spots are retinal hemorrhages and Osler nodes are painful nodes on the digits both caused by immunologic vasculitis. Rapid Review Endocarditis Patient will be complaining of fever, rash, cough and myalgias PE will show Fever, Roth spots, Osler nodes, Murmur, Janeway lesions, Anemia, Nailbed hemorrhages, Emboli (FROM JANE) Diagnosis is made by echocardiography and Duke's criteria Most commonly caused by: IVDA: S. aureus, tricuspid Native valve: Streptococci, mitral Treatment is antibiotics Comments: GI malignancy: S. bovis

Question: What is the cause of decreased partial pressure of oxygen and increased partial pressure of carbon dioxide after posterior packing placement?

Answer: Although not completely understood, it is believed to be caused by the nasopulmonary reflex. Rapid Review Epistaxis Most common source: Anterior bleeds: Kiesselbach's plexus Posterior bleeds: Sphenopalatine artery Treatment is: Anterior bleeding: direct pressure, packing, cautery Posterior bleeding: packing (foley, gauze pack, intranasal balloon device) Admit patients with posterior packing to a m

Question: What is the treatment for the opportunistic infection, Cryptococcus?

Answer: Amphotericin B combined with flucytosine. Rapid Review Pneumocystis Pneumonia (PCP) Patient with a history of HIV Complaining of gradual onset of non-productive cough Labs will show CD4 < 200, increased LDH CXR will show bilateral infiltrates (bat wing pattern) Most commonly caused by Pneumocystis jirovecii Treatment is TMP-SMX

Question: What is the underlying pathophysiology behind anovulatory bleeding?

Answer: Anovulatory bleeding is caused by the failure of the corpus luteal cyst to form leading to absence of progesterone and unopposed estrogen stimulation on the endometrium. Rapid Review Dysfunctional Uterine Bleeding (DUB) MCC of abnormal vaginal bleeding in reproductive women Menarche, perimenopause Anovulatory: ↑ Estrogen, ↓ progesterone → endometrial hyperplasia/bleeding Unpredictable bleeding Ovulatory: Predictable bleeding Dx of exclusion Rx: combination OCPs Unstable bleeding: IV estrogen

Question: What serum test can be used to establish HBV immunity?

Answer: Anti-Hepatitis B surface antigen (anti-HBsAg) antibody is the best marker of HBV immunity. Rapid Review Acute Hepatitis HAV: fecal-oral, shellfish, alone (no carrier), asymptomatic, acute HBV: HBsAg: active and chronic infection Anti-HBs: recovered or immunized Anti-HBc IgM: early marker of infection, positive in window period Anti-HBc IgG: best marker for prior HBV HBeAg: high infectivity Anti-HBeAb: low infectivity HCV: IVDA, chronic, cirrhosis, carcinoma, carrier HDV: dependent on HBV coinfection HEV: fecal-oral (enteric) high mortality rate among pregnant (expectant) patients, epidemics, HAV and HEV are fecal-oral: "The vowels hit your bowels" Autoimmune hepatitis: young females Alcoholic hepatitis: moderate transaminase elevation, AST>ALT Supportive rx

Question: At what age does immune thrombocytopenia present?

Answer: Any age. Rapid Review Idiopathic Thrombocytopenic Purpura (ITP) Patient will be a child 2 - 6 yrs old With a history of recent viral infection Complaining of red spots on skin or easy bleeding PE will show petechiae, purpura, and gingival bleeding Labs will show platelets < 50,000 µL Most commonly caused by antiplatelet antibodies Treatment is observation, steroids, IVIG

Question: Which ethnic group has a high rate of cystic fibrosis and should be recommended for prenatal screening?

Answer: Ashkenazi Jews. Rapid Review Cystic Fibrosis Autosomal recessive mutation in CFTR gene → abnormal Cl transport → thick secretions Presentations: meconium ileus, failure to thrive, persistent cough, lung infections Pseudomonas aeruginosa Broad-spectrum ABX, bronchodilators

Question: How often should sexually active homosexual men be screen for HIV?

Answer: At least annually.

Question: What is the definition of preterm or premature labor?

Answer: Before 37 weeks. Rapid Review Chorioamnionitis Intra-amniotic infection Risk factors: preterm labor, premature rupture of membranes, prolonged rupture of membranes GBS infection ↑ at 18 hrs Rx: ampicillin + gentamicin

Question: What type of spider bite causes a hemorrhagic blister, necrotic ulcer, or target lesion?

Answer: Brown recluse spider.

Question: What antecedent bacterial infection is associated with GBS?

Answer: Campylobacter jejuni infection. Rapid Review Guillain-Barré Syndrome Antecedent pulmonary or GI illness (Campylobacter jejuni) Rapid ascending symmetrical weakness Lower extremity weakness > upper extremity weakness Deep tendon reflex loss → respiratory failure Normal rectal tone CSF: markedly ↑ protein with up to 100 lymphocytes/μL Obtain pulmonary function tests Rx: IVIG or plasmapharesis, possible prophylactic intubation

Question: In patients with chronic or unresponsive paronychia, what organisms are likely involved?

Answer: Candida albicans and atypical mycobacteria. Rapid Review Paronychia Infection of lateral nail fold S. aureus Mild rx: ABX, warm soaks, Abscess rx: I&D

Question: Which NSAID medication would be most appropriate for a patient with peptic ulcer disease due to its lower risk of contributing to gastrointestinal ulceration?

Answer: Celecoxib, or Celebrex®, selectively inhibits cyclooxygenase-2, making it moderately safer to use in patients at risk for gastrointestinal bleeding. Rapid Review Peptic Ulcer Disease Most common cause of UGIB RFs: smoking, H. pylori, NSAIDS, ASA, steroids Free air under diaphragm on upright CXR if perforated Duodenal > gastric Duodenal: awakens patient at night, pain relieved by food Gastric: early satiety, pain immediately after meals Proton pump inhibitors, endoscopy Complications: perforation, gastric outlet syndrome

Question: Vertical nystagmus is always associated with which type of vertigo?

Answer: Central vertigo; peripheral vertigo is associated with horizontal or horizontal rotary nystagmus. Rapid Review Central Vertigo Lesion of brainstem or cerebellum Constant dizziness with variable response to movement Gradual onset Rare hearing loss and Tinnitus Spontaneous nystagmus increased by visual fixation Positional nystagmus- Multidirectional no latency or fatigue N/V more severe in cerebellar hemorrhage Cerebellum=ataxia, lateralizing dysmetria Brainstem=Dysarthria, dysphasia, diplopia, Horner's syndrome, blindness Treat symptoms Patients require admission

Question: What medication should be used in the treatment of RMSF in pregnant patients or those with allergies to tetracyclines?

Answer: Chloramphenicol. Rapid Review Rocky Mountain spotted fever (RMSF) Patient with a history of recently in the woods hiking or camping Complaining of abrupt onset of severe headache, photophobia, vomiting, diarrhea, and myalgia PE will show maculopapular eruption on the palms and soles Diagnosis is made by skin biopsy Most commonly caused by Rickettsia rickettsia Treatment is ALWAYS doxycycline, even in children

Question: What is the definitive treatment of paraphimosis?

Answer: Circumcision. Rapid Review Paraphimosis Inability to pull retracted foreksin back over glans Vascular emergency Initial rx: manual reduction with firm pressure for 5-10 minutes Dorsal slit if reduction unsuccessful

Question: What is the curative treatment for a patient with ulcerative colitis with pancolitis?

Answer: Colectomy. Rapid Review Celiac Disease Patient will be complaining of diarrhea, steatorrhea, flatulence, weight loss, weakness and abdominal distension Labs will show IgA anti-endomysial (AGA) and anti-tissue transglutaminase (anti-tTG) antibodies Diagnosis is made by small bowel biopsy Treatment is gluten free diet Comments: associated with dermatitis herpetiformis (chronic, very itchy skin rash made up of bumps and blisters)

Question: An accumulation of blood in the potential space between the dura and skull best describes which type of head injury?

Answer: Epidural hematoma.

Question: How is erythema toxicum neonatorum diagnosed?

Answer: Erythema toxicum neonatorum (ETN) is diagnosed clinically based on history, physical examination, and peripheral smear of intralesional contents.

Question: How long should treatment continue for patients with acute dystonia?

Answer: Forty-eight hours (treatment typically includes benztropine or diphenhydramine). Rapid Review Dystonic Reaction Antipsychotics, antidopaminergic drugs Hours - days Muscle spasms (i.e. torticollis) Stiffness Rx: diphenhydramine, benztropine

Question: If estrogen is contraindicated in a postmenopausal woman, which alternative medication can be used to treat significant hot flashes, especially if there is concurrent sleep disturbance?

Answer: Gabapentin. Rapid Review Menopause Decreased ovarian follicles --> decreased estrogen Average age: 51 Menopause before 40: premature ovarian failure Increased FSH is specific HAVOCS: Hot flashes, Atrophy of Vagina, Osteoporosis, CAD, Sleep abnormalities Vaginal bleeding: r/o malignancy

Question: What type of injury is most commonly involved in bicycle accidents?

Answer: Head trauma

Question: What staging system is used in the diagnosis of hidradenitis suppurativa?

Answer: Hurley staging system. Rapid Review Hidradenitis Suppurativa: Patient will be a woman With a history of lesions that have waxed and waned over the past few years Complaining of tender nodules in her axillae and anogenital area PE will show lesions that are tender, malodorous, often with exudative drainage, sinus tracts Treatment is intralesional triamcinolone, topical clindamycin Comments: Hurley staging system describes the severity of disease

Question: Which endocrinopathies predispose adolescents to Slipped Capital Femoral Epiphysis?

Answer: Hypothyroidism and growth hormone deficiency. Rapid Review Slipped Capital Femoral Epiphysis Obese African-American male adolescents (11-13) Left > right Limp, hip or groin pain Painful/↓ hip internal rotation AP/lateral X-ray of bilateral hips: abnormal Klein line, Bloomberg's sign Non-weight-bearing, orthopedic consultation

Question: What is the most common cause of respiratory acidosis?

Answer: Hypoventilation/Apnea.

Question: What percentage of vertigo is due to central vestibular etiologies?

Answer: Less than 10%. Rapid Review Meniere's Disease ↑ Endolymph Recurrent vertigo + tinnitus + sensorineural hearing loss Abrupt onset Ear fullness Vestibular rehabilitation, low-salt diet, HCT, surgery

Question: The differential diagnosis of interstitial lung disease (ILD) includes which three major conditions?

Answer: Lung malignancy, lung infection and congestive heart failure. Rapid Review Restrictive Lung Disease ↓ Lung expansion → ↓ FVC and ↓ TLC FEV1/FVC: normal

Question: What are three complications of bacterial sinusitis?

Answer: Meningitis, orbital cellulitis and sinus bone osteitis. Rapid Review Sinusitis MCC: viral URI Pain over sinus Purulent rhinorrhea ABX for refractory cases

Question: What formula is used to calculate the minimum systolic blood pressure for a child age 1-10 years?

Answer: Minimum SBP = 70 + (2 x age in years). Rapid Review Normal Pediatric Heart Rates <1: 100-160 bpm 1-2: 90-150 bpm 2-5: 80-140 bpm 6-12: 70-120 bpm >12: 60-100 bpm

Question: What finding on a Tzank smear confirms the diagnosis of an HSV infection?

Answer: Multinucleated giant cells. Rapid Review Herpes Simplex Labialis Patient will be complaining of painful oral lesions PE will show painful vesicles and erosions on the tongue, buccal mucosa, and lips Labs will show multinucleated giant cells on Tzanck smear Diagnosis is made clinically. Gold standard is tissue culture with polymerase chain reaction (PCR) Most commonly caused by herpes simplex virus (HSV) type 1 Treatment is topical antiviral therapy or oral acyclovir

Question: Why is nitrofurantoin not used in the treatment of pyelonephritis?

Answer: Nitrofurantoin is excreted unchanged in the urine only reaching effective concentrations in the bladder. Rapid Review Cystitis F > M E. coli > S. saprophyticus Dysuria, ↑ frequency, urgency, hematuria Pyuria, bacteriuria, urine leukocyte esterase + Urine nitrites: gram-negative organisms Acute uncomplicated cystitis: TMP-SMX, nitrofurantoin, or fluoroquinolone for 3-5 days Acute uncomplicated cystitis with comorbid conditions: TMP-SMX, nitrofurantoin, or fluoroquinolone for 7 days Beyond The Boards Fluoroquinolones: In 2016, the US Food and Drug Administration (FDA) stated that the serious adverse effects associated with fluoroquinolones generally outweigh the benefits for patients with acute sinusitis, acute bronchitis, and uncomplicated urinary tract infections who have other treatment options. For patients with these infections, fluoroquinolones should be reserved for those who have no alternative treatment options. In most cases of acute sinusitis and acute bronchitis, viral rather than bacterial infections are common, and antimicrobials may not be needed. This announcement was based on an FDA safety review showing that systemic fluoroquinolone use is associated with side effects, which although uncommon can be disabling and potentially permanent, including those involving the tendons, muscles, joints, nerves, and central nervous system.

Question: What medication is contraindicated when using tadalafil?

Answer: Nitroglycerin.

Question: What is the most common treatment of molluscum contagiosum?

Answer: Observation and reassurance of spontaneous resolution. Rapid Review Molluscum contagiousum Poxvirus Umbilicated papules Direct contact with material found within papular lesions allows spread Autoinoculation is common Spontaneous resolution at 6-9 months

Question: What are two relative contraindications to performing an arthrocentesis?

Answer: Overlying cellulitis and coagulopathy. Rapid Review Septic Arthritis Age <35: N. gonorrhea S. aureus most common overall Hematogenous spread Fever, pain, ↓ ROM Knee (most common) Arthrocentesis (WBC >50,000 with >75% PMNs) IV ABX, surgical washout

Question: What is the most common cause of croup?

Answer: Parainfluenza virus. Rapid Review Laryngotracheitis (Croup) Patient will be a non-toxic appearing child, 6 months to 3 years old Complaining of URI symptoms with barky, seal-like cough, inspiratory stridor, low-grade fever Labs will show Steeple sign on PA view Most commonly caused by Parainfluenza virus Treatment is steroids, aerosolized epinephrine

Question: What leads best demonstrate U waves?

Answer: Precordial leads (V1 to V6). Rapid Review Hypokalemia Patient with a history of diuretics use, diarrhea, vomiting Complaining of weakness, hyporeflexia, cramping, paresthesias ECG will show U waves, T wave flattening, ST-depression, QT prolongation Treatment is potassium replacement along with magnesium

Question: Urate crystals come from the breakdown of foods that are rich in what?

Answer: Purines. Rapid Review Gout Patient will be a middle-aged man Complaining of acute onset of pain in the first MTP (Podagra) Labs will show needle-shaped crystal with negative birefringence Most commonly caused by uric acid crystals Treatment is: Acute: NSAID's Chronic: allopurinol or colchicine Comments: can be triggered by loop and thiazide diuretics

Question: What causes hypovolemic hyponatremia with elevated urine sodium >20?

Answer: Renal sodium loss from thiazide diuretics, osmotic diuresis, nephropathy, or mineralcorticoid deficiency. Rapid Review Hyponatremia Serum Na <135 Severity depends on rate of development of symptoms Nausea, vomiting, paresthesias, AMS Central pontine myelinolysis if corrected too fast Hypertonic saline for symptomatic patients

Question: What is the most common location of foreign body aspiration?

Answer: Right mainstem bronchus. Rapid Review Laryngotracheitis (Croup) Patient will be a non-toxic appearing child, 6 months to 3 years old Complaining of URI symptoms with barky, seal-like cough, inspiratory stridor, low-grade fever Labs will show steeple sign on PA view Most commonly caused by Parainfluenza virus Treatment is steroids, aerosolized epinephrine

Question: What is the most common significant dysrhythmia in pediatrics?

Answer: SVT. Rapid Review Supraventricular Tachycardia amended 6/20/16 All tachydysrhythmias that arise above the bifurcation of the bundle of His Characteristics: Atrial rate 120-200 Rhythm: regular Narrow QRS Causes Pre-excitation syndromes (WPW) Mitral disease Digitalis toxicity Drugs and toxins Hyperthyroidism Treatment: Vagal maneuvers (Valsalva) Adenosine (first-line medication), ßBs, CCBs Unstable: synchronized cardioversion

Question: What is the treatment of idiopathic intracranial hypertension?

Answer: Serial lumbar punctures, acetazolamide and diuretics. Rapid Review Idiopathic Intracranial Hypertension (Pseudotumor Cerebri) Young obese females Vitamin A toxicity, steroids, tetracycline ↓ CSF absorption HA + visual sx Papilledema, CN VI palsy ↑ Opening pressure on LP Acetazolamide, serial LPs

Question: What other mechanism can retinal hemorrhages occur in other than abusive head trauma?

Answer: Severe blunt force mechanism such as a motor vehicle collision or fall from a great height. Rapid Review Child Abuse Bucket handle fracture Posterior rib fractures Fractures of different ages Cutaneous: bruises, bites, burns Shaken baby syndrome: retinal hemorrhages

Question: Traumatic retinoschisis along with intracranial or intraocular hemorrhages in a child under 5 should alert you to what possibility of what condition?

Answer: Shaken baby syndrome (Nonaccidental inflicted neurotrauma). Rapid Review Retinal Detachment Patient will be complaining of painless loss of vision, floaters, flashing lights, curtain lowering sensation PE will show retina appears hazy gray with white folds Treatment is stat ophthalmology consult

Question: What red blood cell disorder has a characteristic increase in mean corpuscular hemoglobin concentration (MCHC)?

Answer: Spherocytosis. Rapid Review Iron Deficiency Anemia Microcytic Hypochromic Low MCV Low MCHC Low reticulocyte count Low iron High total iron binding capacity Iron supplementation

Question: What are the best tests for confirmation of Salmonella enteritis?

Answer: Stool cultures and PCR assay. Rapid Review Salmonellosis Patient with a history of eating poultry, meat, or eggs Complaining of fever, bloody diarrhea, and abdominal cramps Labs will show fecal WBCs Comments: common cause of osteomyelitis in children with sickle cell disease

Question: What is the most common cause of epiglottitis?

Answer: Strep. and Staph. species. H. influenzae was the most common prior to the Hib vaccine. Rapid Review Epiglottitis Patient will be complaining of rapid onset of fever and dysphagia PE will show patient leaning forward, drooling, inspiratory stridor Imaging will show "thumbprint" sign Most commonly caused by H. influenzae, Streptococcus Treatment is IV antibiotics and airway management

Question: Why is the hyperthyroidism of subacute thyroiditis typically less severe than that of Grave disease or toxic nodular goiter?

Answer: Subacute thyroiditis hyperthyroidism is primarily due to excessive releases of the less-active free T4, as opposed to the more-active T3 excess in Graves disease.

Question: List some diseases that are associated with Raynaud's phenomenon.

Answer: Systemic lupus erythematosus, mixed connective tissue disorder, rheumatoid arthritis, Sjogren's, dermatopolymyositis, vasculitis and primary pulmonary hypertension. Rapid Review Raynaud's Arteriolar vasospasm Triggered by cold, emotion Fingers become white → blue → red Rewarming, CCBs Primary Raynaud's or Raynaud's disease - idiopathic Secondary Raynaud's or Raynaud's phenomenon - associated underlying disease present

Question: What primary disease process is associated with sterile vegetation endocarditis on both sides of the involved valve?

Answer: Systemic lupus erythematosus. Rapid Review Endocarditis Patient will be complaining of fever, rash, cough and myalgias PE will show Fever, Roth spots, Osler nodes, Murmur, Janeway lesions, Anemia, Nailbed hemorrhages, Emboli (FROM JANE) Diagnosis is made by echocardiography and Duke's criteria Most commonly caused by: IVDA: S. aureus, tricuspid Native valve: Streptococci, mitral Treatment is antibiotics Comments: GI malignancy: S. bovis

Question: What is the classic radiographic abnormality seen in aortic coarctation?

Answer: The "figure-3" sign, characterized by prestenotic aortic dilation, coarctation indentation and poststenotic aortic dilation. Rapid Review Coarctation of the Aorta PE will show higher blood pressure in the arms than in the legs EKG will show LVH CXR will show notching of ribs Diagnosis is made by echo Treatment is balloon angioplasty with stent placement, or surgical correction Comments: Associated with Turner's syndrome

Question: What criteria is used to stage Hodgkin and non-Hodgkin lymphoma?

Answer: The Ann Arbor system. Rapid Review Polycythemia Vera Patient will be complaining of headache, dizziness, pruritus after showering PE will show hypertension, splenomegaly Labs will show increased RBC mass, overproduction of all cell lines, increased Hgb Most commonly caused by mutation of the Janus kinase 2 gene (JAK2) Treatment is phlebotomy, hydroxyurea, aspirin

Question: How is Finkelstein's test performed?

Answer: The examining physician grasps the patient's thumb and ulnar deviates the hand sharply. Rapid Review Carpal Tunnel Syndrome Patient with a history of extensive wrist usage such as typing Complaining of pain and numbness in the first, second, and third digits, especially at night PE will show Phalen's sign: reproduction of symptoms with wrist hyperflexion, Tinel's sign: reproduction of symptoms with percussion over carpal tunnel Most commonly caused by median nerve compression Treatment is NSAIDs, volar splint in neutral position

Question: Why are patients with sickle cell disease at greater risk for infection?

Answer: They have functional asplenia and are at increased risk for infection from encapsulated organisms. Rapid Review Sickle Cell Disease Sickling → vaso-occlusive ischemia ↓ O2, dehydration, acidosis → sickling Newborns: initially asymptomatic (due to ↑ HbF) Most common presentation in infants: dactylitis Aplastic crisis: ↓ Hb + reticulocytopenia, parvovirus B19 Acute chest syndrome Most common cause of death in adults Fever, CP CXR: pulmonary infiltrate Splenic sequestration crisis: rapid splenic sequestration of RBCs → splenomegaly + severe anemia Stroke Aseptic necrosis of the femoral head Dysfunctional spleen → ↑ infection risk Salmonella osteomyelitis S. pneumoniae sepsis: most common cause of death in children

Question: What organism may be identified by a wet mount made from a vaginal swab?

Answer: Trichomonas vaginalis, a flagellated protozoan visualized on the wet mount causing an itchy vaginal discharge with a fishy odor. Rapid Review Ectopic Pregnancy Patient will be a woman With a history of prior ectopic, PID, tubal surgery, IUD Complaining of vaginal bleeding, abdominal pain, amenorrhea PE will show adnexal tenderness or unexplained hypotension Labs will show positive pregnancy test and lower than expected serum ß-hCG levels Diagnosis is made by ultrasound Most commonly located in a fallopian tube Treatment is MTX or surgery

Question: Which class of migraine abortive medications should be avoided in patients with Prinzmetal's angina?

Answer: Triptan medications, which can exacerbate coronary vasospasms, should be avoided in patients with Prinzmetal's angina. Rapid Review Prinzmetal's Angina Intermittent coronary artery vasospasm Risk factors: smoking, cocaine ​ST elevations Negative cardiac biomarkers Rx: nitrates, CCBs

Question: What volume of blood is associated with escalating intracranial pressure to > 30 mm Hg?

Answer: Volumes that exceed 5 ml of blood cause a steep rise in intracranial pressure rising above 30 mm Hg. Rapid Review Cushing's Triad: Pre-terminal findings Suggestive of increasing intracranial pressure with herniation or impending herniation Hypertension, bradycardia, irregular respirations Indicative of ischemia and arterial compression

Question: What is the definition of Stevens-Johnson/toxic epidermis necrolysis?

Answer: When the body surface area involved in 15-30%.

Question: Supraventricular tachycardia is most common in what population?

Answer: Young, healthy individuals with no underlying heart disease.

Which of the following vital signs is considered abnormal in a 1-year-old patient? Heart rate of 160 beats per minute Oxygen saturation of 98% on room air Respiratory rate of 26 breaths per minute Systolic blood pressure of 80 mm Hg

Correct Answer ( A ) Explanation: A heart rate of 160 beats per minute would be considered tachycardia in a 1-year-old patient. The average heart rate for a 1-year-old is 120 beats per minute. An oxygen saturation of 98% on room air (B) is within normal limits for all aged patients, including a 1-year-old. The average respiratory rate for a 1-year-old is 26 breaths per minute (C) and is therefore considered normal. The average systolic blood pressure for a 1-year-old is 90 mm Hg (D) with a minimum systolic blood pressure of 72 mm Hg.

A 28-year-old married heterosexual man presents to your office for his annual exam. He was tested for HIV after getting married and has not had any other sexual partners. He wants to know how often he should be screened for HIV. What prevention guidance do you provide? Based on your clinical judgment, he is at low risk for contracting HIV and therefore does not need repeat screening at this time He should be screened annually, regardless of risk He should be screened every 10 years, regardless of risk He should be screened every 5 years, regardless of risk

Correct Answer ( A ) Explanation: According to the Centers for Disease Control and Prevention (CDC), initial HIV screening should be a normal aspect of clinical care. Individuals ages 13-64 should be routinely screened for HIV infection. Providers may use clinical judgment regarding repeat screening of patients who are low-risk, such as those in a monogamous relationship who were previously screened for HIV and found to be HIV-negative. Patients who report sexual activity with a new partner or who are considering a new sexual relationship should be encouraged to undergo HIV testing. Persons at high risk for contracting HIV including men who have sex with men, injection drug users, sex workers, sex partners of HIV-positive individuals, and heterosexuals with more than one sex partner since their last HIV screening should be screened at least annually (B). There are no guidelines that advise screening at 5-year (D) or 10-year (C) intervals.

A 19-year-old man with a history of sickle cell disease presents to your office with chest pain. He states that over the last few days he has had fever, wheezing, and cough. Given this history, you are most concerned with what condition? Acute chest syndrome Costochondritis Myocardial infarction Pericarditis

Correct Answer ( A ) Explanation: Acute chest syndrome is characterized by fever, chest pain, wheezing, cough, hypoxia, and a new lung infiltrate in at least one whole lung segment in patients with a history of sickle cell disease.. It usually occurs secondary to vaso-oclusion, but also may be due to infarction, embolism, or bacterial pneumonia. It is the most common cause of death in sickle cell disease. The syndrome is more frequent in children (mild course) than adults (more severe course). Costochondritis (B) is typically limited to focal chest wall tenderness and not associated with pulmonary complaints. Myocardial infarction (C) is highly unusual in this age group. Pericarditis is associated with chest pain and fever. Although less likely in this patient with cough and wheezing, an ECG should be obtained. Acute pericarditis (D) is usually a self-limited condition and less concerning than acute chest syndrome.

A 21-year-old woman with no prenatal care presents for evaluation of lower abdominal pain and fever. She estimates that she is approximately 7.5 months pregnant. On questioning, she acknowledges intermittent pain for two days and a gush of fluid shortly after the pain began. Her temperature is 101.8°F. Physical examination is notable for purulent material in the vaginal vault. Which of the following is the most likely diagnosis? Chorioamnionitis Endometritis Pelvic inflammatory disease Urinary tract infection

Correct Answer ( A ) Explanation: Beginning at 16 weeks, the membranes of the chorioamniotic sac adhere to the cervical os and are at risk for infection. Chorioamnionitis is an intra-amniotic infection of the chorion and amniotic layers of the amniotic sac. The placenta and fetal membranes may also be involved. It is caused by an ascending infection of normal vaginal flora. Risk factors include: premature rupture of membranes, preterm labor, prolonged rupture of membranes, multiple vaginal examinations and genital tract infections. Clinical findings include fever, uterine tenderness and maternal and fetal tachycardia. Women may also have purulent vaginal discharge on examination. This is a clinical diagnosis and patients require intravenous antibiotics, most commonly ampicillin and gentamicin. Endometritis (B) in an infection of the uterine endometrium that affects between 2 and 8% of pregnancies. The infection develops on the second or third post-partum day and is characterized by fever, abdominal pain and foul-smelling lochia. Pelvic inflammatory disease (C) does not occur during this stage of pregnancy due to the mucous plug that seals the cervix. It may occur during the first trimester although it is quite rare. A urinary tract infection (D) does not cause systemic signs of infection as described in the patient unless it has moved to the upper urinary tract causing pyelonephritis.

A 32-year-old man presents with a 1-year history of frequent abdominal cramping, nonbloody diarrhea, and a 20-lb weight loss. He has no history of foreign travel, antibiotic use, or consumption of well water. He complains of a chronic, pruritic rash that is vesicular in nature. Which of the following is the most likely diagnosis? Celiac disease Collagenous colitis Crohn disease Irritable bowel syndrome

Correct Answer ( A ) Explanation: Celiac disease is an autoimmune disease triggered by dietary gluten. Clinical presentation ranges from non-bloody diarrhea and failure to thrive, to iron-deficiency anemia or osteoporosis. Serum IgA tissue transglutaminase (TTG) antibodies are highly sensitive and specific for celiac sprue, and a small bowel biopsy showing villous atrophy is the gold standard for diagnosis. Dermaitis herpetiformis almost universally occurs in association with celiac disease. It is characterized by an intense, pruritic papulovesicular rash that occurs in a symmetrical distribution over the extensor surfaces of the arms and limbs. Treatment for celiac disease is aimed at adopting a gluten-free diet, however recurrent cases of dermatitis herpetiforms should be treated with dapsone, in conjunction with a gluten-free diet. Collagenous colitis (B) causes watery intermittent diarrhea with greater than 10 bowel movements per day. Some patients report nocturnal diarrhea, abdominal pain, vomiting, and dehydration, however significant weight loss is not usually seen. Rare complications may include mucosal atrophy with long standing collagenous colitis. On colonoscopy, the mucosa of the colon typically looks normal, but biopsies of affected tissue usually show deposition of collagen in the lamina propia. There is no rash associated with this disease. Crohn disease (C) is an inflammatory bowel disease characterized by transmural inflammation of the gastrointestinal tract. It may involve any part of the gastrointestinal tract but is usually seen in the terminal ileal and perianal locations. It presents with prolonged diarrhea (with or without gross bleeding), abdominal pain, weight loss, and fever. Extraintestinal manifestions, fistulaes and abscesses are some of the common complications. Rare cutaneous malformations such as erythema nodosum and pyoderma gangrenosum are some of the extraintestinal manifestions however these do not have the same presentation of the rash that is seen in celiac disease. Irritable bowel syndrome (D) is a chronic condition that is associated with diarrhea, constipation, abdominal bloating and abdominal pain that is relieved by defecation. Diet modifications and lifestyle changes that reduce stress are used as effective treatments. There is no rash associated with irritable bowel syndrome.

You suspect a cardiac structural defect in a 6-month-old infant with peripheral edema and hypertension. Skin color is normal. A murmur is not appreciated. However, the blood pressure is markedly higher in the arms than legs. Which of the following is the most likely diagnosis? Aortic coarctation Great artery transposition Patent ductus arteriosus Tetralogy of Fallot

Correct Answer ( A ) Explanation: Congenital cardiac defects can be defined into two classifications: cyanotic and acyanotic. The most common acyanotic lesions are coarctation of the aorta, pulmonary stenosis, aortic stenosis, atrial septal defect, ventricular septal defect, atrioventricular canal and patent ductus arteriosus. Coarctation of the aorta is characterized by narrowing of the aorta anywhere along its length but most commonly around the left subclavian artery takeoff. This acyanotic lesion presents with congestive heart failure, cardiomegaly, hypertension and shock. The classic exam finding is blood pressure higher in the arms than legs, and pulses bounding in the arms and decreased in the legs. Rib notching (inferior aspect of the rib) is the classic radiographic finding in patients with coarctation of the aorta. This occurs as a result of dilatation of intercostal arteries. Surgical correction is usually required, and is most commonly performed between 2 and 4 years of age. Tetralogy of Fallot (D) and great artery transposition (B) are cyanotic, not acyanotic, congenital defects. Patent ductus arteriosus (C) can present with symptoms of congestive heart failure. However, the most common finding is a continuous, systolic and diastolic murmur, commonly called a machinery-like murmur. Although the pulse pressure is also commonly widened, the blood pressure is usually not asymmetric between arms and legs.

A seven-year-old boy is involved in a rollover motor vehicle collision. He arrives intubated by the paramedic service. On exam, you note him to be flexing both upper extremities. Which of the following is the most sensitive indicator of herniation? Bradycardia Hypertension Hypotension Irregular respiration

Correct Answer ( A ) Explanation: Cushing's triad is usually a pre-terminal event seen in patients with increased intracranial pressure and cerebral herniation through the foramen magnum. It is associated with decreased level of alertness, hypertension, bradycardia, and irregular respirations. Bradycardia is the first sign and is therefore the most sensitive indicator. Although hypotension (C) can be seen in neurological trauma, it is more commonly seen in neurogenic shock and is not part of Cushing's triad. Hypertension (B) and irregular respirations (D) are part of Cushing's triad, but they are less sensitive and manifest later in the course than bradycardia and are therefore less sensitive.

An overweight 38-year-old man presents with severe left foot pain that began acutely 2 days ago. He denies recent injury and has no history of surgery to the ankle. He has a past medical history of hypertension. His current medications include lisinopril-hydrochlorothiazide. His vital signs are 99°F, BP 148/86, and HR 92. He works as a train conductor. His job requires a lot of jumping on and off the train. Otherwise, he rarely exercises and has a poor diet. He has had 5 sexual partners in the last 12 months, and uses barrier-protection. He had gonorrhea once in his 20s but has no penile discharge currently. He has not had any recent skin infections. He has never used intravenous drugs. He is not having issues with any other joints. He has a family history positive for osteoarthritis in his maternal grandmother and paternal grandfather, but there is no history of rheumatoid disorders. Physical exam reveals a warm great toe that is moderately swollen, erythematous, and extremely tender to the touch at the metatarsalphalangeal joint. What is the most likely diagnosis? Gout Navicular fracture Osteoarthritis Septic arthritis

Correct Answer ( A ) Explanation: Gout is crystal-induced arthropathy caused by needle-like urate crystals. These crystals are formed most commonly in the metatarsalphalangeal joint of the big toe, ankle, wrist, and knee. The crystals develop as purines break down into uric acid. The crystals develop in peripheral joints because these are cooler areas of the body and the crystals are less soluble at lower temperatures. Most purines come from dietary intake of purine-rich foods (e.g., liver, kidney, anchovies, asparagus, consommé, herring, meat gravies and broths, mushrooms, mussels, sardines, sweetbreads). Certain comorbid conditions are associated with a higher incidence of gout, including hypertension, diabetes mellitus, renal insufficiency, hypertriglyceridemia, hypercholesterolemia, obesity, and anemia. Use of diuretics also increases the risk of attacks. Attacks are usually monoarticular, and begin abruptly. Attacks usually reach maximum intensity in 8-12 hours. The affected joint becomes swollen, warm, erythematous and exquisitely tender. Gout is confirmed by identifying uniquely polarized (birefringent) crystals from the joint aspirate. Navicular fractures (B) primarily occur from sports related activities, such as running, or marching. The navicular helps maintain the medial longitudinal arch of the foot, and is important for absorbing the heel strike as well as enabling the push-off during walking. In this type of injury, the pain would be mid-foot rather than in the toe, and it would be unlikely to be accompanied by significant erythema or warmth. Osteoarthritis (C) predominantly involves the weight-bearing joints, including the knees, hips, cervical and lumbosacral spine, and feet. Osteoarthritis most often occurs in those over the age of 55, and is therefore less likely in a 38 year-old male. The physical exam for osteoarthritis includes crepitus and tenderness, but not erythema and warmth as in the above patient. Septic arthritis (D) is an infection in the joint, most often seeded by trauma, surgery, or injection. Risk factors include age greater than 80 years, diabetes mellitus, rheumatoid arthritis, prosthetic joint, recent joint surgery, skin infection, cutaneous ulcers, intravenous drug abuse, alcoholism and previous intra-articular corticosteroid injection. Most cases of septic arthritis present with fever, joint pain, and impaired range of motion, and the symptoms develop slowly over a few days or weeks.

A 24-year-old woman presents to Urgent Care after tripping over her backpack and hitting her head at home. A friend noted brief period of unconsciousness lasting less than 3 seconds. Prior to arrival to urgent care, she vomits once. On your physical exam, she is noted to have a normal physical exam with normal vital signs. According to the Canadian CT Head Rule, which of the following is the most appropriate next step in management? Observation Transfer to emergency department for a CT scan of the head with contrast Transfer to emergency department for a CT scan of the head without contrast Transfer to emergency department for an MRI scan of the head

Correct Answer ( A ) Explanation: Head injuries can be life threatening therefore it is critical to identify those patients at risk for significant pathology. Serious head trauma can result in an epidural hematoma, subdural hematoma, subarachnoid hemorrhage, intracerebral hemorrhage, and skull fracture. For minor head trauma, guidelines help clinicians decide on whether a head CT scan is needed to diagnose or rule out an intracranial injury. One set of rules called the Canadian CT Head Rule devised a set of criteria seen below on which patients with minor head trauma require a head CT scan. The patient in the clinical scenario does not meet any of these criteria and therefore can undergo a period of observation. The patient should be watched closely for signs of increased intracranial pressure (due to expanding hematoma) such as vomiting, confusion, motor or sensory deficits, visual changes, altered consciousness. It should be noted that the Canadian CT Head Rule is a guideline and any time a clinician feels a patient may have an intracranial injury despite not meeting the Canadian CT Head Rule, the clinician should go ahead and get the head CT scan. The patient does not meet any of the high-risk criteria in the Canadian CT Head Rule and can be observed. A CT head scan without contrast (C) is the standard test of choice in patients suspected of having an intracranial injury. CT head scan with contrast (B) is not the recommended study in patients with suspected intracranial hemorrhage since blood and contrast are both hyperdense on CT scanning. Head CT scan with IV contrast is reserved for patients with intracranial lesions such as metastatic brain cancers and abscesses. MRI (D) is not the first line imaging modality for head trauma since it is time consuming and is not superior in identifying bleeding compared to a CT scan in the acute setting.

A 12-year-old girl presents to the emergency department with diffuse abdominal pain, nausea and vomiting. History is significant for polyuria, polydipsia and a seven pound weight loss in the last month. Her labs are notable for a venous blood gas with a pH of 7.2, and electrolytes with a bicarbonate of 15 mEq/L, glucose of 350 mEq/L and a potassium of 3.2 mEq/ L. A bolus of normal saline 20 ml/kg is administered and an insulin drip is started with saline 0.45%. Electrolytes are rechecked in 2 hours and reveal a potassium of 2.8 mEq/L. Which of the following findings is expected to be seen to be seen on this patient's ECG? Flattened T wave Prolonged QRS interval Prolonged QTc Widened P wave

Correct Answer ( A ) Explanation: Hypokalemia (potassium<3.5 mmol/L) is associated with a flattening of the T waves on an ECG. Potassium levels below 2.7 mmol/L are associated with T wave inversion, PR interval prolongation, increased width and amplitude of the P wave, prominent U waves (positive deflection after T wave), and apparent long QT interval from the fusion of the T and U waves. Worsening hypokalemia can lead to supraventricular tachydysarrhythmias, atrial flutter, atrial tachycardia and life-threatening ventricular dysarrhythmias. Individuals with diabetic ketoacidosis (DKA) have profoundly depleted potassium. However, decreased insulin, volume depletion, and acidemia cause potassium to shift from the intracellular to extracellular space. Therefore, peripheral potassium levels will appear normal or high while total body potassium is low. improved hydration, increased renal perfusion and iatrogenic insulin administration, potassium begins to shift back to the intracellular space causing a rapid decrease in measured potassium serum levels. It is important to monitor electrolytes every 2 to 4 hours during the initial management of DKA. When the potassium level is between 4 and 5 mEq/L, potassium replacement should be initiated with potassium chloride, potassium phosphate or potassium acetate. Prolonged QRS interval (B) can be caused by hyperkalemia as well as sodium channel blockade. As mentioned above, severe hypokalemia can appear as an artificial prolonged QT interval as the T and U waves fuse. True prolonged QTc (C) can be found in hypomagnesia and hypocalcemia. A widened P wave (D) can be found in hypokalemia and hyperkalemia but is associated with increased amplitude in hypokalemia and is associated with decreased amplitude in hyperkalemia.

A 16-year old boy with a history of asthma presents to the ED with severe shortness of breath and audible wheezing. He uses an inhaled corticosteroid and a long acting beta-2-agonist at home daily. However, he has had to use his short acting beta-2-agonist roughly every hour for the past day. In the ED, his vital signs are BP 114/72, HR 106, RR 28, oxygen saturation 94% on room air, and temperature 99.0°F. Diffuse wheezing is appreciated and intercostal retractions are observed. After oxygen is initiated, what is the next step in management for this patient? Begin nebulized albuterol Give epinephrine Give systemic corticosteroids Prepare for intubation

Correct Answer ( A ) Explanation: This patient most likely is experiencing an acute asthma exacerbation. Asthma exacerbations clinically present with shortness of breath, wheezing, cough, and chest tightness. Asthma is defined by reversible airway obstruction, airway inflammation, and airway hyperresponsiveness. Triggers of asthma include dust, pollen, cold weather, exercise, and medications such as aspirin or beta-blockers. The first step in treatment of an acute asthma exacerbation includes oxygen and beta-2-agonist nebulizers, such as albuterol. Ipratropium is often added to the first three doses of albuterol in those with a severe exacerbation. Response to treatment is monitored by clinical appearance, physical examination, and peak-flow measurements. A mnemonic for the ED treatment of asthma is: BIOMES - Beta-agonists, ipratropium, oxygen, magnesium sulfate, epinephrine, and steroids. Systemic corticosteroids (C) are usually initiated in the emergency department to decrease airway inflammation, but this should occur following initiation of an albuterol nebulizer. The onset of action of the albuterol is minutes, while corticosteroids take longer. Epinephrine (B) is given to those with refractory, severe asthma. Those with severe asthma may also require terbutaline, magnesium, or heliox. For some patients experiencing a severe asthma exacerbation, intubation (D) may be warranted, but this patient does not require intubation at this time.

Which of the following is the most common manifestation of abusive head trauma in infants? Epidural hematoma Retinal hemorrhage Subarachnoid hemorrhage Subdural hematoma

Correct Answer ( B ) Explanation: Abusive head trauma (previously referred to as shaken baby syndrome) is a form of inflicted head trauma and the leading cause of child abuse fatalities. It is a well-recognized clinical syndrome caused by violent shaking of infants, direct blows to the head, dropping or throwing a child, and asphyxia. It typical occurs in infants younger than 1 year, but may be seen in children up to 3 years old. The classic constellation of abusive head trauma includes subdural hematoma, traumatic brain injury, and retinal hemorrhages. Retinal hemorrhages are present in up to 75% of cases and are virtually pathognomonic. Absence of retinal hemorrhage does not rule out child abuse. The pathophysiology of retinal hemorrhages is uncertain. It is unclear whether bleeding is a result of increased intracranial pressure transmitted to the eye or occurs directly within the eye itself, perhaps through increased pressure along the retinal vein with subsequent disruption of the vessel. Retinal hemorrhages may involve the area in front of the retina (preretinal hemorrhages), the vitreous, and the subretinal space in addition to the retina. Hemorrhages may be described as "dot and blot" hemorrhages or flame or splinter hemorrhages. Epidural hematoma (A), subarachnoid hemorrhage (C), and subdural hematoma (D) all occur in abusive head trauma but less often than retinal hemorrhages do.

A 35-year-old woman is being managed for multiple episodes of chest pain that have been awakening her from sleep on several early mornings for the past 2 months. An electrocardiogram after one episode showed ST elevation. Coronary angiography did not reveal any stenotic lesions. She has no cardiac risk factors and is otherwise healthy. Which of the following daily medications is most likely to provide relief from her condition? Adenosine Amlodipine Aspirin Propranolol

Correct Answer ( B ) Explanation: Amlodipine is the most appropriate selection to use as daily prophylaxis against the anginal pain caused by Prinzmetal (variant) angina. Amlodipine, as well as other long-acting calcium channel blockers and nitrates, is effective at preventing the coronary vasospasm responsible for the chest pain seen in Prinzmetal's angina. This syndrome most commonly occurs in women under 50 years of age, with a classic presentation being angina in the early morning that awakens a patient from sleep. Of note, Prinzmetal's angina is not brought on by typical factors responsible for aggravating the angina of atherosclerosis, and patients may have no coronary risk factors. The work-up of any chest pain of suspected cardiac origin would include an electrocardiogram which, in this case, would show ST-segment elevation in the distribution of the vasoconstricted coronary artery. Coronary angiography will also be helpful to rule out a stenotic lesion in need of intervention. If Prinzmetal angina without a complicated stenotic lesion is diagnosed, patients should be counseled to avoid substances that contribute to vasoconstriction. Two important contributors are cigarette smoking and the use of cocaine. Treatment should consist of daily calcium channel blockers or nitrates to prevent vasoconstriction, and break-through nitrates as needed for acute episodes. Adenosine (A) is an antiarrhythmic medication used most commonly in the management of supraventricular tachycardia. It is not recommended for Prinzmetal's angina, and occasionally even causes chest pain as an adverse effect. Aspirin (C), though commonly beneficial in cardiovascular risk reduction, does not have a role in managing the episodes of vasoconstriction seen in Prinzmetal's angina. Propranolol (D), a beta-blocker, does not have a role in managing Prinzmetal's angina and has been known to contribute to coronary vasospasm. It is generally only considered in patients with Prinzmetal's angina whose symptoms are related to a stenotic lesion.

A 16-month-old boy and his adoptive parents present to your office with a complaint of cough and concern about the patient's weight. The parents indicate that they don't have much information about the patient's birth parents and since adopting him four months ago they have noticed daily shortness of breath, cough and wheezing. He is also a fussy eater and is in the second percentile of weight on the growth chart. Which lab finding is most consistent with the diagnosis? Elevated exhaled nitric oxide testing Elevated sweat chloride Elevated white blood cell count Positive skin allergy test

Correct Answer ( B ) Explanation: Cystic fibrosis (CF) is an autosomal recessive disease that occurs predominantly in Caucasians. Multiple organ systems are affected because of a mutation in the cystic fibrosis transmembrane conductance regulator protein, a regulatory protein found in exocrine tissues. Of the patients who survive the neonatal period, 90% have pulmonary complications. Historically patients were diagnosed with cystic fibrosis after presenting with symptoms. Now newborn screening allows for detection of the disease before patients become symptomatic. Clinical presentation includes meconium ileus, failure to thrive and respiratory symptoms. Patients with CF have a shortened lifespan and end-stage lung disease is generally the cause of death. Clinical symptoms, a positive newborn screening or a sibling with the disease are reasons to test for CF. Initial evaluation is with sweat chloride testing and an elevated result should be referred for DNA testing to detect CF-related mutations. Exhaled nitric oxide testing (A) may reveal decreased levels in children with cystic fibrosis but is not a first-line diagnostic test for suspected CF. Elevated white blood cell count (C) occurs with infection. Unless the patient with CF also has an infection, this result would not be seen on a blood test of a patient with CF. Skin allergy test (D) is not part of the workup for a patient with suspected CF.

Which of the following patients with Salmonella enteritis should receive antibiotics? 13-month-old with diarrhea and no signs of volume depletion 13-year-old boy with sickle cell disease 19-year-old man living in a college dorm 8-year-old boy with no medical problems

Correct Answer ( B ) Explanation: Empirical antibiotic therapy for Salmonella enteritis is not recommended for otherwise healthy patients without severe disease. However, it is recommended in certain groups including patients with sickle cell disease. Salmonella is the most common documented bacterial enteritis in the US. The bacteria is acquired from ingestion of contaminated food, typically beef and poultry. The disease presents with fever, loose, watery stool that may have blood or mucous in them and colicky abdominal pain. Empirical antibiotic therapy is typically not required in healthy patients. Antibiotics do not shorten the duration of the disease and may increase the length of time that a patient is a carrier of the bacteria. Antibiotics are recommended (although not proven to be effective) based on severity of disease, age, immune system competency and other comorbidities. Additionally, health care workers or other people who may spread the disease should be treated to eradicate the carrier state. Treatment options include ciprofloxacin, norfloxacin and azithromycin. Antibiotics are recommended for infants <12 months (A) and for those >50 years of age (D). There are no specific recommendations for patients living in dormitories (B).

A 67-year-old man presents to your office six months after prostatectomy for follow up on his previously diagnosed prostate cancer. Labwork reveals an elevated serum prostate specific antigen. After discussing options with the patient, a decision is made to use pharmacologic treatment. Which of the following is the most appropriate initial agent? Doxazosin Leuprolide Tadalafil Terazosin

Correct Answer ( B ) Explanation: In the United States, prostate cancer is the most frequently diagnosed type of cancer in men after skin cancer. Prostate cancer is seen more commonly in African-American men and the likelihood of developing this type of cancer increases with age. Risk factors include a family history of prostate cancer, cigarette smoking and a diet high in animal fat. Most patients diagnosed with prostate cancer are asymptomatic and the cancer is found on digital rectal exam (DRE) or because of an elevated serum prostate specific antigen (PSA). Diagnosis is made with biopsy. Treatment is based on severity of disease and after discussion with patient since options may significantly impact quality of life. Initial treatment discussion should focus on the patient's life expectancy and the characteristics of the tumor. Treatment options for localized prostate cancer include active surveillance, prostatectomy, radiation therapy and hormonal therapy. When systemic therapy is indicated, androgen deprivation therapy is initiated using gonadotropin releasing hormone (GnRH) agonists such as leuprolide. Doxazosin (A) and Terazosin (D) are alpha-1-adrenergic antagonists used in the treatment of benign prostatic hypertrophy. Tadalafil (C) is a phosphodiesterase-5 inhibitor used to treat both erectile dysfunction and benign prostatic hypertrophy.

A 60-year-old man presents to your office complaining of generalized pruritus following warm showers. He also states that he has been fatigued and had several nosebleeds in the past month. Physical exam reveals splenomegaly and engorged retinal veins. You suspect polycythemia vera. Which of the following findings will confirm that your diagnosis? Bence Jones protein JAK2 mutation Philadelphia chromosome Reed-Sternberg cells

Correct Answer ( B ) Explanation: JAK2 mutation is present in nearly all cases of polycythemia vera. The mutation causes the bone marrow to make too many blood cells. Common presenting complaints include headache, dizziness, tinnitus, blurred vision, fatigue, epistaxis (due to mucosal engorgement and platelet dysfunction), erythromelalgia (burning pain in feet or hands with erythema, pallor, and cyanosis) and pruritus following warm showers. The pruritus is due to histamine from increased basophils. Physical exam findings include plethora, engorged retinal veins, splenomegaly, and hypertension. It is important to distinguish polycythemia vera from reactive erythrocytosis, which can be caused by hypoxia, smoking, COPD, high altitude, sleep apnea, and renal and liver disease. Treatment includes serial phlebotomy and medical management (hydroxyurea, aspirin, allopurinol). Thrombosis is the most common complication of this disorder and may cause death. Transformation to acute myelogenous leukemia (AML) occurs in in small number of individuals. Bence Jones protein (A) is the hallmark finding in multiple myeloma, a malignancy of hematopoietic stem cells which causes abnormal plasma cells. Philadelphia chromosome (C) is found in chronic myeloid leukemia (CML), a myeloproliferative disorder that causes overproduction of white blood cells. Patients often present with complaints of fatigue, night sweats and low grade fevers. Reed-Sternberg cells (D) are found in Hodgkin lymphoma. Patients often present due to a painless mass in the neck.

A 50-year-old man presents with episodic severe vertigo lasting hours, with associated symptoms of unilateral tinnitus, fluctuating low-frequency hearing loss, and aural fullness. Which of the following is the most likely diagnosis? Benign paroxysmal positional vertigo Meniere's disease Perilymph fistula Vestibular neuronitis

Correct Answer ( B ) Explanation: Meniere's disease is characterized by episodic severe vertigo lasting hours, with associated symptoms of unilateral tinnitus, fluctuating low-frequency hearing loss, and aural fullness. Typical onset is in the fifth decade of life. The cause is uncertain but is speculated to result from allergic, infectious, or autoimmune injury. The histopathologic finding includes endolymphatic hydrops, which is thought to be caused by either overproduction or underresorption of endolymph in the inner ear. Meniere's disease is a clinical diagnosis mostly based on history. Testing may be obtained to support the diagnosis and rule out other disorders. Audiometry often demonstrates a low-frequency sensorineural hearing loss. An FTA-ABS test may be obtained to rule out syphilis. Electronystagmography (ENG) may demonstrate a unilateral peripheral vestibular weakness on caloric testing. When the diagnosis is uncertain, a brain MRI with contrast is obtained to evaluate for a retrocochlear lesion. The differential diagnosis of Meniere's disease includes acute labyrinthitis, neurosyphilis, labyrinthine fistula, autoimmune inner ear disease, vestibular neuronitis, and migraine-associated vertigo. The most common cause of peripheral vestibular vertigo in adults is benign paroxysmal positional vertigo (A). BPPV occurs in all age groups but more often between ages 50 and 70 but is not associated with hearing loss and made worse with movement. In a perilymph fistula (C) rapid changes in air pressure (barotrauma), otologic surgery, violent nose blowing or sneezing, head trauma, or chronic ear disease may cause leakage of perilymph fluid from the inner ear into the middle ear and result in episodes of vertigo. Associated signs and symptoms are variable but can include a sudden pop in the ear followed by hearing loss, vertigo, and sometimes tinnitus. Acute vertigo associated with nausea and vomiting (but without neurologic or audiologic symptoms) that originates in the vestibular nerve is known as vestibular neuronitis (D). Vestibular neuronitis can occur spontaneously or can follow viral illness.

A woman with systemic sclerosis complains of painful blue fingers. It only seems to affect her during the cold winter months, especially if she doesn't wear gloves. She doesn't complain of arthralgias, myalgias or pruritus. Her most recent chest radiograph shows bibasilar fibrosis. Her last four serum creatinine results were normal. Which of the following medications do you recommend? Lisinopril Nifedipine Prednisone UVA-1 phototherapy

Correct Answer ( B ) Explanation: Systemic sclerosis has a multitude of manifestations. One of the more common signs is Raynaud's phenomenon. This cold temperature-induced condition results in intermittent and recurrent digital vasoconstriction, which causes finger pallor, cyanosis or hyperemia, as well as pain, numbness, ischemia and possibly ulceration. Treatment is geared at avoiding cold temperatures, wearing gloves and thick socks and smoking cessation. Medications can also be used, and include aspirin, prostaglandins, vasodilators and calcium-channel blockers, like nifedipine, especially when there is concomitant pulmonary fibrosis. Angiotensin converting-enzyme inhibitors, like lisinopril (A), are recommended in patients with systemic sclerosis who also have renal insufficiency. This patient's kidneys are most likely not affected by her primary condition. Corticosteroids, like prednisone (C), are recommended for myalgia or weakness in a patient with systemic sclerosis and suspected or documented myositis. UVA-1 phototherapy (D) is used to treat systemic sclerosis associated pruritus, not vasomotor abnormalities.

A 3-day-old female developed a rash 1 day ago that has continued to progress and spread. The infant was born at term after an uncomplicated pregnancy and delivery to a healthy mother following excellent prenatal care. The infant was discharged 2 days ago in good health. She does not appear to be irritable or in distress, and she is afebrile and feeding well. On examination, abnormal findings are confined to the skin, including her face, trunk, and proximal extremities, which have macules, papules, and pustules that are all 2-3 mm in diameter. Her palms and soles are spared. A stain of a pustular smear shows numerous eosinophils. Which one of the following is the most likely diagnosis? Acne neonatorum Erythema toxicum neonatorum Herpes simplex virus keratitis Staphylococcal pyoderma

Correct Answer ( B ) Explanation: This infant has a typical presentation of erythema toxicum neonatorum. Erythema toxicum neonatorum (ETN) is a benign self-limited eruption occurring primarily in healthy newborns in the early neonatal period. Erythema toxicum neonatorum is characterized by macular erythema, papules, vesicles, and pustules, and it resolves without permanent sequelae. Acne neonatorum (A) consists of closed comedones on the forehead, nose, and cheeks. Staphylococcal pyoderma (D) is vesicular and the stain of the vesicle content shows polymorphonuclear leukocytes and clusters of gram-positive bacteria. Because the mother is healthy and the infant shows no evidence of being otherwise ill, systemic infections such as herpes (C) are unlikely.

A 35-year-old woman presents for several days of anterior neck pain radiating to her ears, accompanied by dysphagia and "restlessness." She endorses a low-grade fever and fatigue that is "lingering" from her recent flu-like illness. Physical exam shows a tender, symmetrically enlarged thyroid. Laboratory studies show a normal complete blood count, suppressed thyroid stimulating hormone, low antithyroid antibody titers, and a high erythrocyte sedimentation rate. In addition to propranolol, the most appropriate treatment plan includes which of the following medications? Ampicillin Aspirin Levothyroxine Methimazole

Correct Answer ( B ) Explanation: This patient has subacute thyroiditis which is treated with oral propranolol to manage her symptoms of hyperthyroidism and aspirin to manage pain. Subacute thyroiditis, or deQuervain thyroiditis, is a common, transient condition that usually presents after a recent viral illness, such as an upper respiratory infection. Young and middle-aged women are the typical population. Symptoms include an acute, painful enlargement of the thyroid gland with dysphagia, fever, and fatigue. Pain often radiates to the jaw or ears. Approximately 50% of patients will develop symptoms of thyrotoxicosis, including palpitations, diaphoresis, vomiting or diarrhea, and a high fever. Laboratory findings will include a suppressed TSH, elevated free T4 levels, and markedly elevated erythrocyte sedimentation rate. Symptoms of hyperthyroidism can remain for several weeks and may be followed by a 4-6 month period of hypothyroidism. Normal thyroid function generally restores within 12 months. Long-term thyroid replacement medication is required in 5% of patients. Ampicillin (A) given parenterally may be an appropriate choice in suppurative thyroiditis. However, suppurative thyroiditis is distinguished from sub-acute thyroiditis by an elevated leukocyte count, which is not present in this patient. Levothyroxine (C) is the necessary treatment for hypothyroidism, not hyperthyroidism. Subacute thyroiditis usually allows of return of normal thyroid function within 12 months and levothyroxine is rarely needed. Methimazole (D) is a treatment option for hyperthyroidism, such as that caused by Graves disease. However, the hyperthyroidism of subacute thyroiditis will typically resolve spontaneously in 4-6 weeks; a beta-blocker is usually sufficient to manage these temporary hyperthyroidism symptoms.

A 44-year-old woman presents with pain to her second digit for 2 days. Physical examination reveals a small fluctuant area at the eponychium. What management is indicated? Acyclovir x 7 days Incision and drainage Oral antibiotics and warm soaks X-ray of the finger

Correct Answer ( B ) Explanation: This patient presents with a paronychia or a localized abscess of the nail fold that should be incised and drained. Patients will present with swelling and tenderness of the soft tissue at the lateral nail fold. Staphylococcus aureus is the most common causative agent in adults but in children, anaerobes may be more present (secondary to finger sucking). Management is with incision and drainage. The finger should be soaked in warm water and the edge of the skin at the nail should be gently lifted away from the nail using a scalpel blade. Subsequently, the cavity should be irrigated and packing gauze should be placed under the eponychium for 24 hours. If this were a herpetic whitlow, acyclovir (A) would be appropriate. Incision and drainage should be avoided for a herpetic whitlow as this can spread the infection. Oral antibiotics and warm soaks (C) are helpful if only cellulitis is present without abscess. X-ray of the finger (D) will not be helpful in the diagnosis or workup of a paronychia.

A 21-year-old woman presents with urinary frequency and dysuria for 2 days. Vital signs are within normal limits. Examination reveals mild suprapubic tenderness with no costovertebral angle tenderness. What treatment is indicated? Ceftriaxone Nitrofurantoin Non-contrast CT scan of the abdomen and pelvis Phenazopyridine

Correct Answer ( B ) Explanation: This patient presents with an uncomplicated acute cystitis and can be treated with nitrofurantoin. Urine tract infection (UTI) is an inflammatory response to microorganisms in the urinary tract. Patients will present with dysuria, frequency, urgency, hematuria and suprapubic pain when infection is in the bladder. Cystitis represents infection of the lower urinary tract (pyelonephritis is infection of the upper urinary tract) and is common in women of all age groups but relatively uncommon among young men. Uncomplicated cystitis is defined as inflammation of the bladder in patients in the absence of pregnancy, medical comorbidities or toxic appearance. The most common causative organisms are Escherichia coli and Staphylococcus saprophyticus. First line antibiotic agents for uncomplicated cystitis include nitrofurantoin and trimethoprim-sulfamethoxazole (TMP-SMX). Ciprofloxacin is a common first line drug in regions where resistance to TMP-SMX is greater than 10-20%. Ceftriaxone (A) is a third generation cephalosporin with broad coverage of gram negative pathogens and is widely used for the treatment of pyelonephritis and complicated urinary tract infections. A non-contrast CT scan of the abdomen and pelvis (C) is useful in investigating the presence of renal stones but this patient's history is incompatible with this disease. Phenazopyridine (D) also known as pyridium is used to alleviate the pain, irritation, discomfort, or urgency caused by a cystitis but it does not treat the infection. Its most common side effect is urine discoloration.

A 32-year-old woman presents with vaginal bleeding for 2 weeks. She states she has about 1 pad of bleeding every 2-3 hours. Vital signs are stable and physical exam only reveals blood from the cervical os. The patient's hemoglobin is 12 g/dl and her pregnancy test is negative. What treatment is indicated for this patient? Admission for dilation and curettage Combination oral contraceptives Hysterectomy Intravenous estrogen therapy

Correct Answer ( B ) Explanation: This patient presents with non-life threatening dysfunctional uterine bleeding (DUB), which can initially be managed with combination oral contraceptives. DUB is typically split into anovulatory (90%) and ovulatory (10%). In patients with vaginal bleeding of childbearing age, the most important first step in diagnosis is to rule out pregnancy. After this, it is important to explore other causes including medications, genital tract pathology and systemic disease. Once these are excluded, a diagnosis of DUB can be reached. Some treatments include NSAIDs that inhibit PGE1 production and can both relieve cramping and pain and also decrease bleeding. In anovulatory bleeding, combination oral contraceptive pills can aid in regulating the menstrual cycle and counteract the effects of unopposed estrogen. Typically, patients are instructed to take combination oral contraceptive pills twice a day for 5-7 days or until the bleeding stops followed by once daily dosing. Dilation and curettage (A) is typically offered to patients with heavy vaginal bleeding evidenced by hemodynamic instability. A hysterectomy (C) is rarely needed in the treatment of DUB but is indicated for patients with heavy bleeding and hemodynamic instability in which conservative management fails. Intravenous estrogen therapy (D) is effective in stopping heavy bleeding, but is not considered first line therapy.

What is the causative agent for the above rash? Coxsackie virus Herpes simplex virus Smallpox virus Staphylococcus aureus

Correct Answer ( B ) Explanation: This patient presents with painful grouped vesicles on an erythematous base consistent with a herpes simplex virus (HSV) infection. HSV is a common viral infection affecting a considerable portion of adults. There are two variants of HSV that cause human infection. HSV-1 typically affects nongenital sites and HSV-2 typically affects the genital area and is transmitted by sexual contact. HSV lesions typically present as painful, grouped vesicles that are localized in a nondermatomal distribution. Oral lesions are the most common site of HSV-1 infections. In addition to affecting the lips, they can also cause ulcerations to the gingiva, tongue and mouth. Outbreaks can be accompanied by fever and lymphadenopathy. A first episode of HSV can be treated with acyclovir, famciclovir or valacyclovir for 7-10 days. These agents accelerate healing and reduce the length of viral shedding. Coxsackie virus (A) causes hand, foot and mouth disease. These lesions are also vesicular and can be confused with HSV but they typically are not grouped and will have lesions on the hand and foot. Smallpox (C) causes diffuse vesicular lesions. Staphylococcus aureus (D) is the common causative organism of impetigo, which presents as lesions with a yellow-gold crust.

An 18-year-old man presents with lower extremity weakness for 4 days. He states that he had a bout of diarrhea 2 weeks ago that resolved but then began feeling weak in his legs. Over the last 24 hours, he has begun to feel weak in his arms and has difficulty breathing. Physical examination reveals 3/5 strength in both lower extremities and 4/5 strength in both upper extremities along with decreased deep tendon reflexes. What management is indicated for the suspected diagnosis? Ceftriaxone and vancomycin Intravenous immunoglobulin MRI of the brain Oral steroids

Correct Answer ( B ) Explanation: This patient presents with symptoms and signs concerning for Guillain-Barre syndrome (GBS) and should be treated with intravenous immunoglobulin (IVIG). GBS is a peripheral nerve disorder with an unpredictable and heterogeneous clinical course. The most common form is the acute inflammatory demyelinating polyneuropathy (AIDP) accounting for 90% of cases. Most patients present with symptoms days to weeks after the resolution of a gastrointestinal or upper respiratory illness. Symptoms are worse in the lower extremities than in the upper extremities and typically, weakness in ascending. The descending version is called the Miller Fisher variant. In addition to the progressive, symmetric weakness, patients will have variable sensory findings and decrease in deep tendon reflexes. Notably, anal sphincter tone is spared. Respiratory compromise is common and approximately 33% of patients will require intubation and mechanical ventilation. The mortality and recurrence rate are both 3%. All patients presenting with symptoms concerning for GBS should have vital capacity and negative inspiratory pressure (NIF) measured frequently as a decrease predicts the need for intubation. Lumbar puncture typically shows elevated protein with a mild pleocytosis. Treatment should be started with either IVIG or plasma exchange. GBS is not the result of an ongoing infection so treating with antibiotics (A) will not help. MRI of the brain (C) does not add to diagnosis or management as this is a disease of peripheral nerves. Oral steroids (D) have been shown to delay recovery.

A 21-year-old woman with a history of intravenous drug abuse presents with fever, dyspnea, cough and chest pain. Examination reveals an ill-appearing woman with track marks on both upper extremities. A chest X-ray reveals no infiltrate and urinalysis is unremarkable. What management is indicated? Discharge with a prescription for azithromycin for a respiratory infection Draw blood cultures and admit for intravenous antibiotics and transthoracic echo Draw blood cultures and admit for intravenous antibiotics and valvuloplasty Start antibiotics for community acquired pneumonia and admit

Correct Answer ( B ) Explanation: This patient's presentation is concerning for infective endocarditis and should have a minimum of three blood cultures drawn, started on intravenous antibiotics and ordered for a transthoracic echocardiogram (TTE). If the TTE is negative, the patient, since she is high risk (IVDA), should undergo a transesophageal echocardiogram (TEE). Infective endocarditis is a life-threatening disorder that most frequently occurs in patients with specific risk factors including a history of rheumatic heart disease, prosthetic cardiac valves and intravenous drug abuse (IVDA). There are a number of causative organisms for bacterial endocarditis but staphylococcus (42%) and streptococcus species (40%) are the most predominant. Staphylococcus species are most common in patients with IVDA. Additionally, the valve involved differs based on population factors as well. Patients with IVDA are more likely to develop right-sided (tricuspid and pulmonic valve) bacterial endocarditis. Patients with right-sided endocarditis typically have an acute presentation with fever, and respiratory symptoms as well as chest pain. Septic emboli can travel to the lung causing multifocal pneumonia but peripheral septic embolic events are rarely seen (as opposed to left-sided endocarditis). Right-sided endocarditis is often misdiagnosed as pneumonia due to the presence of respiratory symptoms. Additionally, less than 35% of patients with IVDA and endocarditis will present with a murmur. In this population, an unexplained fever should raise concern for bacterial endocarditis. TEE is the diagnostic modality of choice as it is highly sensitive and specific for diagnosing infective endocarditis. In patients without prosthetic valves, the negative predictive value of TEE approaches 100%. In patients with native valves and a history of IVDA, vancomycin can be started as single treatment. Although the patient has a predominance of respiratory features, the presence of fever and IVDA makes endocarditis a distinct possibility and the patient should not be discharged with the diagnosis of a respiratory infection (A). Valvuloplasty (C) uses a balloon catheter to open a stenotic valve. This is most often performed for aortic stenosis. Although this patient may one day require a valvuloplasty, it is not performed in the early stages of endocarditis. Fever, cough and chest pain may be consistent with pneumonia and early in the clinical course, the chest X-ray may not reveal an infiltrate. However, this patient is at high-risk for infective endocarditis and treatment for pneumonia (D) would not be appropriate.

A 65-year-old man presents to the ED complaining of persistent dizziness. He complains of nausea and feels like the room is spinning. He has a history of hypertension. Vital signs are within normal limits. Which of the following physical exam findings supports a central cause of his dizziness? Hearing loss Limb ataxia Patient remains steady when eyes are open and closed when performing the Romberg test Spontaneous nystagmus that is suppressed by visual fixation Sudden onset of intense, intermittent episodes of disequilibrium

Correct Answer ( B ) Explanation: Vertigo is the perception or sensation of movement. This may be described by the patient as a feeling of swaying, spinning, whirling, leaning, or tilting. The patient may also report feeling intoxicated. Nausea and vomiting may also be seen. Vertigo results from dysfunction in the vestibular system from either its peripheral or central components. Physical examination will reveal nystagmus in nearly all patients, often lateralizing to the affected side. Although less common than peripheral etiologies, central causes of vertigo are more concerning and include conditions such as vertebrobasilar insufficiency, brainstem and cerebellar infarct or hemorrhage, basilar artery migraine, and degenerative diseases such as multiple sclerosis. In general, when compared to peripheral vertigo, the symptoms of central vertigo are less acute and more persistent and may be associated with neurologic deficits. However, exceptions exist, particularly in those patients with cardiovascular risk factors. Limb ataxia is not seen with peripheral causes and is usually attributable to cerebellar lesions. A noncontrast head CT is a reasonable screening test with suspected central vertigo (and is most appropriate to evaluate for potential intracranial hemorrhage). Secondary to the fact that it does not permit adequate visualization of the cerebellum, it is not considered the definitive test. Magnetic resonance imaging with angiography is the study of choice in most cases of suspected central causes of vertigo. Hearing loss (A) and tinnitus associated with vertigo indicates inner ear dysfunction, involving the cochlea, labyrinths, or the vestibulocochlear nerve (CN VIII). Disorders include benign paroxysmal positional vertigo (BPPV), Menière's disease, vestibular neuronitis, CN VIII tumors (i.e., acoustic neuromas), cerebellopontine angle tumors, and ototoxicity from drugs. The Romberg test (C) assesses input from a combination of neural systems. This test is performed on a standing patient with the feet together, the arms to the sides, and the eyes open. The Romberg test evaluates integration of visual, sensory, and vestibular inputs with respect to orientation in space. If the patient remains steady with the eyes open, the systems can be assumed to be intact. If the patient then becomes unsteady once the eyes are closed, it means that the position sense from the extremities is not reaching the CNS or there is vestibular dysfunction. Visual fixation will reduce the intensity of nystagmus (D) in peripheral vertigo. In central vertigo, visual fixation does not affect the degree of nystagmus. Sudden onset of intense, episodic symptoms (E) is common with peripheral causes, whereas central causes are associated with more continuous, less intense symptoms.

A 27-year-old healthy man presents with facial pain and low grade fever. For the last 2 days he has felt congested and noticed green drainage from his nose. Which of the following is the most appropriate management? Antihistamines CT scan of the sinuses Ibuprofen Systemic antibiotics

Correct Answer ( C ) Explanation: Acute sinusitis is defined as inflammation of the nasal cavity and paranasal sinuses lasting less than four weeks. The most common etiology of sinusitis is a viral infection with acute bacterial sinusitis responsible for only 0.5 to 2 percent of episodes. The most predictive signs of acute sinusitis are purulent rhinorrhea, nasal congestion and facial pain. Viral sinusitis and bacterial sinusitis are indistinguishable clinically and expert consensus recommends considering a bacterial cause once symptoms last more than 7 to 10 days. Treatment of sinusitis is aimed at symptomatic control. Once symptoms persist for more than 7 to 10 days, a bacterial etiology is possible and antibiotics should be considered. The use of topical glucocorticoids may confer some benefit although data are mixed in trials. Oral decongestant therapy also does not have significant evidence in support of their generalized use. In cases where eustachian tube dysfunction contributes to the development of sinusitis, oral decongestants appear to have benefits. The use of non-steroidal anti-inflammatory medication like ibuprofen does provide pain relief and should be prescribed to the patient. Antihistamines (A) are often prescribed to patients with sinusitis in an attempt to dry up nasal secretions. However, this benefit has not been demonstrated in trials. A CT scan of the sinuses (B) is not necessary unless there is concern for a complication of sinusitis. Acute sinusitis is a clinical diagnosis and treatment is initiated based on the clinical examination. Systemic antibiotics (D) are not indicated in the management of acute sinusitis until the symptoms persist beyond 7 to 10 days. However, studies suggest that bacterial sinusitis resolves without antibiotics in up to 70% of cases.

An 18-month-old boy presents to the emergency department with two days of cough, low-grade fevers, and congestion. Today the cough has become hoarse and barking. What is the most likely diagnosis? Bronchiolitis Community acquired pneumonia Croup Laryngeal foreign body aspiration

Correct Answer ( C ) Explanation: Croup is an infection of the larynx and trachea and is most commonly caused by parainfluenza virus. It is most common in children 6 to 36 months of age. The illness typically begins as classic symptoms of an upper respiratory tract infection, such as coryza, cough, and fever. Symptoms may then progress to a barking cough, hoarseness, and inspiratory stridor due to upper airway obstruction. Stridor may occur solely with activity or with both activity and rest. Stridor that occurs at rest indicates significant upper airway obstruction. Bronchiolitis (A) is inflammation of the bronchioles due to viral infection, most commonly Respiratory Syncytial Virus. While children with bronchiolitis begin with symptoms of upper respiratory congestion and cough, they do not exhibit the classic hoarseness or barking cough associated with croup. In fact, nasal and chest congestion is prominent features of bronchiolitis, and as such, the cough is often wet sounding. Community acquired pneumonia (B) is a common pediatric diagnosis. Symptoms may begin as a classic upper respiratory tract infection before progressing to signs of lower respiratory tract infection. Hoarseness and a barking cough are not classic features of Community Acquired Pneumonia. Laryngeal foreign body aspiration (D) causes acute choking, respiratory distress, and stridor. In contrast to some bronchial foreign body aspirations, the symptoms of laryngeal foreign body aspiration are abrupt and obvious. In the above patient, the progression of symptoms from upper respiratory infection to barking cough is much more typical of croup than a laryngeal foreign body aspiration.

Which of the following treatments decreases the need for repeat medical visits for children with croup? Codeine Cool mist therapy Glucocorticoids Nebulized racemic epinephrine

Correct Answer ( C ) Explanation: Glucocorticoids are a mainstay of treatment for children with mild, moderate, and severe croup. They provide a long-lasting anti-inflammatory effect and have been shown to decrease croup scores six hours after administration. Additionally, in meta-analyses, glucocorticoids have been shown to decrease the rate of repeat medical visits, emergency room visits, emergency room length of stay, hospital length of stay, need for administration of epinephrine. Dexamethasone is the glucocorticoid most commonly used for treatment of croup due to the necessity for only a single dose. It can be administered orally, intramuscularly, or intravenously. Efficacy is comparable between oral and intramuscular dexamethasone. Codeine (A) is a cough suppressant with sedative side effects. It has not been proven beneficial in croup or any other childhood respiratory illness. Additionally, accurate assessment of mental status may be difficult to assess in children on codeine because of its sedative properties. Cool mist therapy (B) has not been shown to have a long-term impact on the course of croup. However, it may be effective supportive therapy in the child's home. Nebulized racemic epinephrine (D) is used as rescue treatment for children with moderate to severe croup but has not been shown to have an impact on the patient's respiratory status more than two hours after administration.

An 18-year-old, obese woman presents to your office with a complaint of redness and pain in her left axilla. Physical exam reveals a solitary nodule, approximately 2 cm in size, with surrounding inflammation and erythema. She tells you that she's had similar "boils" in the past and that her mother had the same skin condition, which required surgery approximately 10 years after diagnosis. Which of the following is the most appropriate therapy? Finasteride Infliximab Topical clindamycin Topical clotrimazole

Correct Answer ( C ) Explanation: Hidradenitis suppurativa (HS) is a chronic inflammatory skin condition caused by a defect in the follicular epithelium. It is a progressive, disabling condition causing scarring, keloids, contractures, and immobility. Risk factors for the development of HS include obesity, genetics, smoking, diet, and mechanical stress on the skin. Onset occurs in adolescence or adulthood in previously healthy individuals. Diagnosis is a clinical one based on the appearance of the lesions, recurrence, and distribution. Initial treatment is with medical management and first-line treatment for stage I HS is localized treatment with topical clindamycin. Systemic medication or surgery is indicated for patients with severe or refractory cases. Finasteride (A) is a 5-alpha-reductase inhibitor used in the treatment of benign prostatic hyperplasia, male pattern baldness, and off-label for female hirsutism. Small case studies have shown effectiveness in treating HS; however, it should be avoided in women of childbearing age. Infliximab (B) is a disease-modifying antirheumatic drug used in cases of severe or refractory HS. Topical clotrimazole (D) is an antifungal that does not play a role in the treatment of HS.

A patient is found to be hyponatremic. Laboratory evaluation reveals low serum osmolality, urine sodium concentration >20 mmol/L and a fractional excretion of sodium (FENa) >1%. He appears to be "fluid overloaded." Which of the following is the most likely cause of this hyponatremia? Cirrhosis Congestive heart failure Hypertensive nephropathy Syndrome of inappropriate ADH release (SIADH)

Correct Answer ( C ) Explanation: Hyponatremia is defined as sodium less than 135 mEq/L. Hyponatremia can occur in a hypovolemic, euvolemic, or hypervolemic state. Hypervolemic hypo-osmolar hyponatremia is is associated with fluid overload. The etiology is usually from a perceived low intravascular volume by the kidneys and active water reabsorbtion in excess to sodium retention. If urine sodium is low (<20) causes include liver failure, cirrhosis, hepatorenal syndrome, nephrotic syndrome, and CHF. If urine sodium is high (>20) causes include acute or chronic renal failure, such as that caused by hypertensive nephropathy. Treatment of hypervolemic hypo-osmolar hyponatremia is dialysis. Cirrhosis (A) and congestive heart failure (B) is often the cause of hypervolemic hypo-osmolar hyponatremia when the urine sodium is low (<20). SIADH (D) results in euvolemic hyponatremia with urine osmolality greater than serum osmolality. The excess ADH causes total body water to increase thereby diluting total body sodium. Despite the increased total body water, these patients typically do not show evidence of edema or heart failure as the increased water is intracellular not intravascular.

A 25-year old man presents to your office after recently being diagnosed with HIV infection at the health department. You obtain blood work and note that his CD4+ count is 180. This patient should receive prophylaxis against which one of the following opportunistic infections? Histoplasma capsulatum Mycobacterium avium-intracellulare complex Pneumocystis jirovecci Toxoplasma gondii

Correct Answer ( C ) Explanation: Patients with HIV infection and severe immunodeficiency are at risk for certain opportunistic infections. Susceptibility to opportunistic infections can be measured by CD4+ T lymphocyte counts. Patients with a CD4+ count < 200 should receive trimethoprim/sulfamethoxazole for prevention of Pneumocystis jirovecci pneumonia (PCP pneumonia). Pneumocystis jiroveci pneumonia remains relatively common in patients with HIV infection, and may be the presenting manifestation of HIV in patients who have not yet been diagnosed. Patients with P. jiroveci pneumonia classically present with fever, progressive exertional dyspnea, and nonproductive cough. Although there are a wide variety of radiologic findings, chest radiography typically shows bilateral interstitial infiltrates. Toxoplasma-seropositive patients who have a CD4+ T-lymphocyte count of less than 100 should be administered prophylaxis against Toxoplasma gondii (D) encephalitis. The daily double-strength tablet of trimethoprim/sulfamethoxazole is recommended, as the preferred regimen for Pneumocystis prophylaxis appears to be effective against toxoplasmic encephalitis as well and is therefore recommended. Adults and adolescents who have HIV infection should receive chemoprophylaxis against disseminated Mycobacterium avium-complex (B) disease if they have a CD4+ T-lymphocyte count of less than 50. Preferred prophylaxis is clarithromycin or azithromycin. Prophylaxis with itraconazole may be considered for Histoplasma capsulatum (A) in patients with CD4+ T-lymphocyte counts less than 100 who are at especially high risk because of occupational exposure or who live in a community with a hyperendemic rate of histoplasmosis (10 or more cases per 100 patient-years).

An 8-year-old boy presents to your office with complaints of fever, headache and rash. His mother tells you that he recently returned from visiting relatives in North Carolina. Physical exam reveals a maculopapular rash on the patient's wrists and ankles that appears to be spreading to his trunk. His temperature is 102°F (38.9°C). Which of the following is the most appropriate therapy? Amoxicillin Cephalexin Doxycycline Penicillin

Correct Answer ( C ) Explanation: Rocky Mountain spotted fever (RMSF) is a tick-borne illness most commonly found in the Southeast, South Central and certain parts of the Northeastern United States. It is the most common cause of tick-borne fatalities in the United States. RMSF is often described as "the great imitator" of other diseases because of its diverse clinical presentation. The classic triad of headache, fever and a rash is seen in most patients by the second week of infection. Patients may also present with myalgias, central nervous system symptoms such as confusion and lethargy, or gastrointestinal symptoms such as nausea, vomiting, diarrhea or abdominal pain. RMSF is a clinical diagnosis made when exposure to ticks is determined. Doxycycline is the drug of choice to treat RMSF for both adults and children. Doxycycline is also used to treat ehrlichiosis, Lyme disease, and relapsing fever, which can all be confused with RMSF. In 1997, the American Academy of Pediatrics endorsed the use of doxycycline in children with RMSF because of the potential for fatalities. Short courses in children were not found to cause dental staining. Amoxicillin (A), cephalexin (B) and penicillin (D) are all beta-lactam antibiotics. Beta-lactam antibiotics do not provide coverage for RMSF and are therefore not recommended.

A 5-year-old boy presents to your office with rapidly developing sore throat, high fever, restlessness, lethargy and a muffled voice.The patient appears uncomfortable and is in a sitting position leaning forward. A lateral extended neck radiograph shows evidence of a "thumbprint" sign. Which of the following is the most likely diagnosis? Airway foreign body Croup Epiglottitis Peritonsillar abscess

Correct Answer ( C ) Explanation: Signs and symptoms of epiglottitis include rapidly developing sore throat, high fever, restlessness, and lethargy. A "supraglottic," muffled voice is common. Many patients have difficulty with their saliva and drool. Classically, these patients are in a sitting position leaning forward, because this position tends to alleviate obstructive symptoms from the supraglottic swelling. They may show signs of "air hunger" or may have stridor. Epiglottitis (or "supraglottitis") is a condition that requires prompt attention by the physician. Epiglottitis results from bacterial (and rarely viral) infection of the supraglottic structures (epiglottis and arytenoid cartilages). A high level of suspicion is necessary to make a diagnosis and avoid significant morbidity. Patients may rapidly decompensate due to airway compromise. The physician should always be suspicious when a patient presents with fever, sore throat, and difficulty swallowing. Croup, tonsillitis, peritonsillar abscess, and other neck infections may be incorrectly diagnosed in these patients. Epiglottitis occurs mainly in children age 2 to 7 years, although infants, older children, and adults can be affected. Mortality rates of 6% to 7% have been reported in adults. The incidence has significantly decreased since the use of the Hib vaccine. Most cases are now seen in the adult population. Airway foreign body (A) is not associated with a muffled voice or high fever and does not include the thumbprint sign. Croup (B) is subglottic and occurs gradually and is accompanied by a "barky," seal-like cough. Peritonsillar abscess (D) is associated with fever, sore throat for 3 to 5 days, dysphagia, odynophagia, and a muffled, "hot potato" voice. Trismus is common. Examination confirms asymmetric tonsils and peritonsillar edema and erythema and is not associated with the thumbprint sign on radiography.

A 23-year-old woman complains of headaches and arthralgias for the past 3 days. She presents to urgent care because of a painful rash on her face that has progressed into multiple vesicles. This rash is characterized by erythematous macules with dark purple centers with multiple vesicles in different stages. On physical exam there is also mucosal edema within the oral cavity. She recently completed an antibiotic course for a urinary tract infection. Which of the following is the most likely diagnosis? Erythema multiforme minor Staphylcoccal scalded skin syndrome Stevens-Johnson syndrome Toxic epidermal necrolysis

Correct Answer ( C ) Explanation: Stevens-Johnson syndrome is an immune-complex-mediated hypersensitivity reaction that typically involves the skin and the mucous membranes. After one to three weeks of exposure to the causative agent, a prodrome of fever, malaise, headache, cough, and conjunctivitis develops. Skin lesions appear one to three days after this prodrome. The lesions initially appear as erythematous macules with dark purpuric centers, then form atypical target lesions with central dusky purpura or a central bulla, with surrounding macular erythema. Mucosal involvement occurs in almost all affected patients. By definition, Stevens-Johnson syndrome affects less than 10% of the body surface area. Approximately 50% of cases of Stevens-Johnson syndrome are drug induced. Common causative agents include sulfa drugs, antiepileptic drugs, antibiotics, and nonsteroidal anti-inflammatory drugs. Steven-Johnson syndrome is a clinical diagnosis. The causative drug and any unnecessary medications should be discontinued. Aggressive management should include fluid resuscitation, nutritional supplementation, and wound care. Antibiotics should be used only if there is evidence of infection. Intravenous immune globulin has been incorporated into some treatment protocols. Complications include hypotension, renal failure, corneal ulcerations, anterior uveitis, erosive vulvovaginitis, respiratory failure, seizures, and coma. Staphylococcal scalded skin syndrome (B) most commonly occurs in infants and in young children, faint, orange-red, macular erythema with cutaneous tenderness. Periorificial and flexural accentuation may be observed. Characteristic tissue paper-like wrinkling of the epidermis is followed by the appearance of large, flaccid bullae in the axillae, in the groin, and around the body orifices. Toxic epidermal necrolysis (D) is a potentially life-threatening dermatologic disorder characterized by widespread erythema, necrosis, and bullous detachment of the epidermis and mucous membranes, resulting in exfoliation. By definition, toxic epidermal necrolysis affects more than 30 percent of the body surface area. Erythema multiforme minor (A) represents a localized eruption of the skin with minimal or no mucosal involvement. The papules evolve into pathognomonic target lesions that appear within a 72-hour period and begin on the extremities. Lesions remain in a fixed location for at least 7 days and then begin to heal.

Which of the following is most characteristic of a patient with idiopathic intracranial hypertension? Abnormal CSF chemistry analysis Opening pressure of 15 cm H2O on lumbar puncture Transient visual loss Unilateral papilledema

Correct Answer ( C ) Explanation: The exact pathophysiology of idiopathic intracranial hypertension (fomerlly referred to as pseudotumor cerebri or benign intracranial hypertension) is not well understood but suspected to result from an imbalance between the production and reabsorption of cerebrospinal fluid (CSF). It is most commonly seen in young, obese women of childbearing age. Additional risk factors include the use of oral contraceptives, anabolic steroids, tetracyclines and vitamin A. Headache is the predominant condition of the disease. The headache is usually generalized and made worse by maneuvers that impair cerebral venous return (e.g. Valsalva and bending forward). Visual complaints occur commonly and patients may have transient visual disturbances several times a day. After these episodes, periods of visual loss can occur. These episodes likely occur secondary to ischemia of the visual pathways. On physical examination, the patient may have an enlarged blind spot and loss of peripheral vision. Rarely, a sixth nerve palsy occurs. Abnormal CSF chemistry analysis (A) is not typical for idiopathic intracranial hypertension. Most often the analysis is normal, although protein can be low. The opening pressure on lumbar puncture is elevated due to the intracranial hypertension. An opening pressure of 20 cm of H2O would be considered the upper limits of normal. In patients who are obese, a pressure >25 cm H2O is considered abnormal and in non-obese patients, a pressure of >20 cm H2O. Therefore a pressure of 15 cm H2O (B) would be considered normal. Papilledema (D) is often seen, but occurs bilaterally because of the elevated intracranial pressure.

A 58-year-old woman who works on an assembly line complains of bilateral wrist pain for the last several months. She describes pain, numbness, and paresthesias in her thumb, index, and long fingers. Which of the following tests is most likely to be positive? Adson's test Finkelstein's test Phalen's test Tinel's sign

Correct Answer ( C ) Explanation: The patient has carpal tunnel syndrome. Symptoms of carpal tunnel syndrome include gradual onset of numbness, paresthesias, and pain in the thumb, index, and long fingers. Symptoms are often worse at night and after strenuous activity. Carpal tunnel syndrome is commonly caused by repetitive strain. It can also be seen after distal radius and carpal bone fractures and dislocations, or as a result of systemic conditions (e.g rheumatoid arthritis, hypothyroidism, pregnancy, and diabetes). Phalen's test has a sensitivity of 76% and specificity of 80% for carpal tunnel syndrome. The test is performed by having the patient fully flex the wrists and push them together with the hands facing downward. A positive test is elicited if the patient develops paresthesias or numbness in the median nerve distribution within 60 seconds. Tinel's sign is less sensitive that Phalen's sign for carpal tunnel syndrome. A positive test is demonstration of pain or paresthesias in the median nerve distribution when the median nerve is tapped on at the wrist. Cervical radiculopathy and thoracic outlet syndrome should also be considered as symptoms can mimic carpal tunnel syndrome. Nerve conduction studies are used to confirm the diagnosis of carpal tunnel syndrome, with reported sensitivity of 85 to 90%. Nonoperative management for carpal tunnel syndrome includes splinting the wrist in a neutral position, antiinflammatory medications, and cortisone injections into the carpal tunnel. For persistent or severe symptoms, surgical release of the flexor retinaculum is performed. Adson's test (A) is used to assess for thoracic outlet pathology. Finkelstein's test (B) is pathognomonic for de Quervain's tenosynovitis. Tinel's sign (D) is less sensitive for carpal tunnel syndrome than Phalen's test.

A 65-year-old woman presents with a nosebleed. She is tachycardic on presentation. Examination reveals brisk bleeding from the left nares. An anterior pack is placed but the patient continues to bleed profusely into her posterior pharynx. What treatment is indicated? Admit to ENT for further management Place an anterior pack in the right nares Remove anterior pack and place a posterior pack Remove the anterior pack and replace it

Correct Answer ( C ) Explanation: The patient presents with brisk hemorrhage from the nose with a failed anterior pack suggestive of a posterior nasal bleeding source. The initial management of any patient presenting with epistaxis should focus on the airway and assessment of hemodynamic status. This should be followed by application of direct pressure and localization of the bleeding cause. Anterior epistaxis results from bleeding from Kiesselbach's area, which can be directly visualized during evaluation. Posterior bleeding usually originates from the nasopalatine branch of the sphenopalatine artery, which cannot be visualized directly. In a patient where a properly placed anterior pack does not control bleeding, placement of a posterior pack should be performed. In the Emergency Department, this is typically performed by placing a Foley catheter into the nares and advancing it into the posterior pharynx followed by inflation of the balloon and application of anterior traction to tamponade bleeding while waiting for ENT to arrive. After placement of a posterior pack, patients should be given prophylactic antibiotics to avoid the development of sinusitis and toxic shock (via colonization of packing material). Patients with posterior packs must be admitted to the hospital and likely require intensive care management. Posterior packing has been shown to decrease partial pressures of oxygen and increase the partial pressure of carbon dioxide. Dysrhythmias, bradycardia and aspiration have been described after posterior packing. After a properly placed anterior pack fails to control bleeding, placement of a contralateral pack (B) is unlikely to contribute to hemostasis. Similarly, replacing the anterior pack (D) is unnecessary and will delay definitive management. Although the patient will require admission (A) this does not obviate the need to control bleeding emergently in the Emergency Department.

A 12-year-old boy presents with penile pain. On exam you note the image above. What management is indicated? Direct application of ice to the skin Follow up with urology Manual reduction Topical clotrimazole

Correct Answer ( C ) Explanation: The patient presents with paraphimosis, a urologic emergency, requiring emergent reduction of the foreskin. Paraphimosis occurs when the foreskin cannot be returned to its anatomic position covering the glans penis. It can be caused by infection, trauma or hair tourniquets. Subsequent venous congestion can lead to arterial compression, penile necrosis and gangrene. The typical presentation is of erythema and engorgement of the penis distal to the obstruction and proximal flaccidity. Treatment is directed at reducing swelling and edema of the foreskin and reducing it to the neutral position. Parenteral anesthesia or a local nerve block may be necessary for manual reduction. Circumferential compression should be held starting at the glans to reduce edema. This should be followed by manual reduction by placing steady pressure on the glans with both thumbs while holding the shaft straight. If these attempts fail, urology should be consulted for dorsal slit procedure. Patients may be discharged home after reduction as long as they are voiding without difficulty and have no signs of severe infection. Although cold water (in a glove or bag) can be useful in reducing edema, ice (A) should not be directly applied. Urology follow up (B) is necessary after reduction has occurred but patients should not be sent home with phimosis because of the risk of serious complications. Topical clotrimazole (D) is used to treat fungal infections of the glans like balanitis.

A lethargic 17-year-old woman is brought into the Emergency Department by a friend after being found with an empty pill bottle at her bedside. An initial arterial blood gas shows the following: pH 7.28, pCO2 55 mm Hg, HCO3 24 mEq/L. Which of the following acid-base disturbances is present? Acute (Compensated) Primary Metabolic Alkalosis Acute (Uncompensated) Primary Metabolic Acidosis Acute (Uncompensated) Primary Respiratory Acidosis Uncompensated Mixed Metabolic/Respiratory Acidosis

Correct Answer ( C ) Explanation: This arterial blood gas (ABG) shows an abnormally low pH (acidosis) with an elevated pCO2 and a normal Bicarbonate level, consistent with an uncompensated primary respiratory acidosis. Normal values are: pH 7.36 - 7.44, pCO2 36 - 44 mm Hg, HCO3 22 - 26 mEq/L. The primary disturbance will either be respiratory (change in pCO2) or metabolic (change in HCO3). The body will attempt to compensate by retaining or excreting CO2 (if metabolic) or HCO3 (if respiratory). For example, in the above case, the kidneys will increase levels of HCO3 in order to counteract the elevated amount of CO2 in the blood. Since this is not seen (HCO3 is normal), it can be assumed that this disturbance is acute and uncompensated. Acute (compensated) primary metabolic alkalosis (A) is characterized by a normal pH, high pCO2, and elevated HCO3. Acute (uncompensated) primary metabolic acidosis (B) is characterized by a low pH, normal (or low if starting to compensate) pCO2, and low HCO3. An uncompensated mixed metabolic/respiratory acidosis (D) is characterized by a low pH, high pCO2, and a low HCO3.

A 22-year-old man presents to the ED with clonus of his neck to the right. Which of the following drugs is he most likely to be taking? Benztropine Cocaine Haloperidol Ziprasidone

Correct Answer ( C ) Explanation: This man is having a dystonic reaction, which is a common side effect of haloperidol and other typical antipsychotic drugs. Haloperidol is a high-potency antipsychotic that blocks dopamine-2 receptors at the basal ganglia, which can lead to acute dystonia shortly after drug initiation (50% occur within 48 hours, 90% within five days) and sustained movement disorders (parkinsonism and tardive dyskinesia) with prolonged use. Symptoms of acute dystonia include intermittent, involuntary, and uncoordinated hyperkinetic movements most often affecting the tongue, face, neck, trunk, or extremities. Treatment is with IM or IV benztropine or diphenhydramine, and recovery is rapid after medication administration. Benztropine (A) has both anticholinergic and antihistamine activity and is commonly used for the treatment of movement disorders. Although cocaine (B) does not typically cause dystonia, it can increase the risk for dystonic reactions and is associated with choreoathetoid movements that are referred to as "crack dancing." Ziprasidone (D) is an atypical antipsychotic drug that is much less likely to cause dystonia and other extrapyramidal symptoms

A 23-year-old woman presents with pain in in the suprapubic area and right lower quadrant of her abdomen that began 4 hours ago. She denies vagina discharge and vaginal bleeding. Her vital signs are BP 115/65, HR 60, RR 12, and T98.1°F. Her last menses was one week ago. Which of the following diagnostic tests should be obtained first? Complete blood count Gonorrhea and chlamydia DNA amplification Urinalysis Urine beta-hCG

Correct Answer ( D ) Explanation: All women of childbearing age presenting with pelvic complaints require a pregnancy test. Even though this patient reports that she had her menses one week prior to her presentation, ectopic pregnancy and even a normal pregnancy can be associated with vaginal bleeding that can be mistaken for normal menses. Since ectopic pregnancy is a life-threatening condition, it is important to rule it out early in this patient's workup. A urine beta-hCG is a rapid test that has a very high sensitivity that can rapidly determine if a woman is pregnant. A negative urine beta-hCG effectively rules out ectopic pregnancy. In an actively bleeding patient, a complete blood count (A) can provide useful information about the hemoglobin and platelet levels. A low hemoglobin level may prompt a blood transfusion. The evaluation for gonorrhea and chlamydia with DNA amplification (B) may be part of the evaluation of the patient with acute pelvic pain as a helpful adjunct in the work-up of pelvic inflammatory disease. However, in most institutions these results are not available in a timely fashion and do not have the same level of urgency as the exclusion of pregnancy with a beta-hCG test. A urinalysis (C) may also be helpful in the evaluation of a woman with pelvic pain, however, it is not entirely specific to the cause of pain. Urine may be affected by other diagnoses including cervicitis, vaginitis, pelvic inflammatory disease, nephrolithiasis, or appendicitis.

A 12-year-old obese boy presents with groin and proximal anterior thigh pain. He has no pain below the knee. Physical examination shows a significant decrease in internal rotation of the affected hip. Radiographs show a widened growth plate. Which of the following is the most likely diagnosis? Acute transient synovitis Legg-Calve-Perthes disease Osgood-Schlatter disease Slipped Capital Femoral Epiphysis

Correct Answer ( D ) Explanation: Atraumatic pediatric hip pain or limp is usually caused by acute transient synovitis, Legg-Calve-Perthes disease (LCP) or Slipped Capital Femoral Epiphysis (SCFE). SCFE is a fracture in the physis (growth plate) of the femoral head which leads to slippage of the overlying epiphysis. It usually occurs during adolescent growth spurts. Predisposing factors include male sex, obesity and increased sports activities. The typical age range is 10-14 years for girls and 11-16 years for boys. Patients present with pain in the anterior proximal knee or thigh that is exacerbated by activity. On exam, there is loss of hip internal rotation, particularly with the hip flexed. Patients typically walk with the involved extremity externally rotated. AP and frog-leg radiographs show "ice cream falling off the cone." All cases warrant urgent orthopedic evaluation for stabilization surgery. Patients should be nonweight bearing and restricted to bed rest until then. Complications of untreated disease include chondrolysis and osteonecrosis. Acute transient synovitis (A) is considered the most common cause of childhood hip pain or limp. It is most common in boys aged 3-6 years. Radiographs are normal. Treatment includes NSAIDs and relative rest. Legg-Calve-Perthes (B) disease is avascular osteonecrosis of the femoral head. It occurs more commonly in boys aged 4-10 years. Radiographs show a small, sclerosed femoral head and widened joint space. Osgood-Schlatter (C) disease is a common adolescent condition in those active in sports. Repeated microtrauma occurs at the apophyseal cartilage between the anterior tibial tubercle and the secondary ossification center of the patellar tendon. These patients present with anteroinferior knee or superior shin pain and tender tibial tuberosities.

A nine-month-old girl is seen in clinic for a routine child visit. She can pull to stand, play peek-a-boo, copy sounds and gestures of others, and she clings to parents. She eats cereal and table food. You discuss car seat safety with her parents. Which of the following is the best advice to give her parents? Front seat is much safer than the rear seat for children Frontal airbags appear to offer good protection to children in crashes Side airbags do not pose a risk for children who are in the front seat Use of child restraint devices can be expected to reduce fatalities by 71 percent

Correct Answer ( D ) Explanation: Discussion of car seat safety should be part of the routine well child visit. Injuries to passenger vehicle occupants are the predominant cause of motor vehicle deaths among children and adolescents. Proper restraint use in vehicles is the single most effective method for preventing serious or fatal injury. Use of child restraint devices, infant car seats, and booster seats can be expected to reduce fatalities by 71 percent and the risk of serious injuries by 67 percent in this age group. All 50 states and the District of Columbia have laws mandating their use. Children weighing less than 20 pounds may use an infant seat or be placed in a convertible infant-toddler child-restraint device. The infants younger than two years or those weighing less than manufacturer's weight limit should be placed in the rear seat facing backward. Older toddlers and young children can be placed in the rear seat in a forward-facing child harness seat, until it is outgrown. Emphasis must be placed on the correct use of these seats. This includes placing the seat in the right direction, routing the belt properly, and ensuring that the child is buckled into the seat correctly. Children younger than 13 years should never sit in the front seat, especially if an airbag is present. Inflating airbags can be lethal to infants in rear-facing seats and to small children in the front passenger seat. Frontal airbags appear to offer good protection to children in crashes (B) is a wrong statement because frontal airbags offer little protection and also present a risk of serious or fatal injury from the airbag itself. Front seat is much safer than the rear seat for children (A) is false. The rear seat is clearly much safer than the front seat for both children and adults. One study of children younger than 15 years found that the risk of injury in a crash was 70 percent lower for children in the rear seat compared with those sitting in the front seat. Side airbags do not pose a risk for children who are in the front seat (C) is not correct because side airbags do pose a risk. The safest place for children is in the rear middle seat, properly restrained for their age and size.

Which of the following treatments is contraindicated in the treatment of a 5-month-old with supraventricular tachycardia? Adenosine Ice bag to face Propranolol Verapamil

Correct Answer ( D ) Explanation: Due to poor calcium reserves in the sarcoplasmic reticulum in infants, verapamil (calcium channel blocker) use in infants can cause profound hypotension and cardiovascular collapse. Therefore, it should be avoided in patients younger than 12-months-old. Adenosine (A) is the first-line agent to treat supraventricular tachycardia. Applying ice to the face (B) or other vagal maneuvers can be attempted before administration of adenosine. Propranolol (C) is a nonselective beta-blocker used in supraventricular tachycardia, but it is less effective than adenosine.

Which of following statements regarding nitrofurantoin is correct? It can be safely prescribed to neonates for treatment of urinary tract infections It can be safely prescribed to patients with G6PD deficiency It is considered effective therapy for treatment of pyelonephritis It is considered effective therapy for treatment of uncomplicated cystitis

Correct Answer ( D ) Explanation: Nitrofurantoin is bactericidal and interferes with cell wall synthesis. The exact mechanism of action is poorly understood. It is effective for the treatment of uncomplicated lower urinary tract infections caused by E. coli, S. saprophyticus, and E. faecalis. Although effective in the lower urinary tract, nitrofurantoin has poor tissue penetration and low serum levels. Therefore it should not be used to treat pyelonephritis (C) or prostatitis. It is considered pregnancy category B, however is contraindicated in the third trimester and neonatal period (A), as neonatal immature red blood cells are damaged by nitrofurantoin causing hemolytic anemia. Nitrofurantoin can also cause hemolytic anemia in individuals with G6PD deficiency (B).

A patient with significant dyspnea presents for evaluation. You order spirometric testing and obtain results consistent with restrictive pulmonary disease. Which of the following findings is most consistent with restrictive pulmonary disease? Alveolar destruction Bronchoconstriction Excess mucus production Parenchymal abnormalities

Correct Answer ( D ) Explanation: Pulmonary diseases can be classified as obstructive and restrictive. The obstructive conditions, asthma, emphysema, chronic bronchitis, bronchiolitis and bronchiectasis, exist in a state of increased airway resistance, increased compliance and decreased parenchymal recoil. These conditions result in a normal or slightly increased forced vital capacity (FVC), and a decreased forced expiratory volume in the first second of expiration (FEV1) to FVC ratio. On the other hand, restrictive conditions result in decreased airway compliance and increased recoil. This leads to a decrease in the FVC and FEV1 but a near normal FEV1/FVC ratio. The restrictive pulmonary conditions include the interstitial lung diseases (ILD) (parenchymal abnormalities) and the chest wall disorders. Interstitial lung disease includes idiopathic interstitial pneumonia (mostly idiopathic pulmonary fibrosis), drug and radiation induced (iatrogenic) pulmonary fibrosis, hypersensitivity pneumonitis, collagen vascular disease, pneumoconiosis and sarcoidosis. Chest wall restrictive lung disease includes pleural disease, severe obesity and some neuromuscular disorders, such as Guillain-Barre syndrome. Destruction of the alveolar walls (A), especially from abnormal elastic tissue in the airway walls, is the main pathologic process in the obstructive pulmonary disease emphysema. Bronchoconstriction (B) is a key underlying pathologic process in asthma, which is an obstructive, not restrictive, lung disease. Mucus hyperproduction (C) and bronchial wall edema are the main findings in chronic bronchitis, which is classified as an obstructive pulmonary disease.

Which of the following indices reported in a standard complete blood count panel is most helpful to differentiate between iron deficiency anemia and thalassemia? Mean corpuscular hemoglobin (MCH) Mean corpuscular hemoglobin concentration (MCHC) Mean corpuscular volume (MCV) Red blood cell distribution width (RDW)

Correct Answer ( D ) Explanation: Red blood cell distribution width (RDW) is a measure of the deviation in volume of the RBCs. It is calculated by dividing the standard deviation of the MCV by the mean MCV and multiplying by 100. RDW = (SD MCV/mean MCV) x 100. It is useful in differentiating thalassemia from iron deficiency anemia. The RDW is usually increased in iron deficiency and normal in thalassemia. Mean corpuscular hemoglobin (MCH) (A) is a measure of the hemoglobin content within a RBC and is calculated by dividing the hemoglobin concentration by the red blood cell count. It is usually low in both iron deficiency and thalassemia anemia. Mean corpuscular hemoglobin concentration (MCHC) (B) is a measure of the concentration of hemoglobin within the RBC. It is calculated by dividing the hemoglobin concentration by the hematocrit. It is expected to decrease in both iron deficiency and thalassemia anemia. Mean corpuscular volume (MCV) (C) is a measure of RBC size. It is also expected to decrease in both thalassemia and iron deficiency anemia.

A 48-year-old man was hit in the right eye with a softball last night and comes to your office today complaining of new onset of objects floating through his vision along with continuous light flashes and reduced brightness in that eye. Exam reveals full extraoccular movement in all directions. He does complain of mild pain around the eye. Based on these findings, which of the following is the most likely diagnosis? Corneal abrasion Lens dislocation Optic neuritis Retinal detachment

Correct Answer ( D ) Explanation: Retinal detachment can be observed after blunt eye injury, especially in older adults. Retinal detachment may also occur spontaneously, especially in patients with high myopia. The patient may complain of unilateral blurred vision described as a "curtain coming down" over the eye. There is also reduced overall brightness in the involved eye and continuous light flashes, indicating retinal traction. An increase in previous floaters or the onset of new floaters may also occur. After eye trauma, it is imperative to inspect not only the central portions of the retina, but the peripheral portions as well. This examination should be performed in a dark room after instillation of a short-acting mydriatic agent. Any questionable findings should be referred to an ophthalmologist immediately. Corneal abrasions (A) cause patients to complain of a foreign body sensation in the eye and pain with blinking. It is not associated with visual floaters or flashes of light. Optic Neuritis (C) is associated with a painful loss of vision and loss of color vision. Patients with lens dislocation (B) may complain of decreased vision, edge glare, diplopia, streaks of light, haloes, photosensitivity, and ghost images. An abnormal appearing lens will be seen on slit lamp exam.

A six-year-old presents with his mother who is concerned about a new rash. What is the etiology of the lesions pictured above? Autoimmune Bacterial Fungal Viral

Correct Answer ( D ) Explanation: The condition seen in the case above is molluscum contagiosum. This is a common condition caused by a DNA poxvirus. Lesions are raised, umbilicated papules and often appear in crops of five to 10. It is more common in swimmers and wrestlers due to dermal exposure. Patients with atopic eczema are especially at risk. Spread is via contact with the waxy material found in the central core of the lesion. Autoinoculation is common. Lesions typically persist for six to nine months then spontaneously resolve. Lesions may become irritated and superinfected. Extrusion of the central core debris is curative. Other treatment options include tretinoin 0.1%, cantharidin or liquid nitrogen. Since spontaneous resolution is common, observation is most commonly used. Though many autoimmune (A) conditions have dermatologic findings, this raised papular lesion is most consistent with molluscum. Other infectious signs such as fever often accompany bacterial lesions (B). Fungal entities (D) are often raised, dry, and scaly in appearance with well-defined borders.

A three-year-old girl was brought by her parents to the ED because of a possible ingestion. The girl was caught with an open bottle of amitriptyline although the mother is not sure how many tablets were missing. The girl did not experience any mental status changes, vomiting, abdominal pain, or fever. She arrived at the ED within 60 minutes of her possible ingestion. On examination, she is alert, active, with tachycardia, dry mucous membranes, and pupils 5-6 mm. Which of the following is the antidote for this type of ingestion? Flumazenil N-acetylcysteine Naloxone Sodium bicarbonate

Correct Answer ( D ) Explanation: The girl possibly took amitriptyline, which is a tricyclic antidepressant (TCA). Cardiovascular and CNS symptoms dominate the clinical presentation of TCA toxicity. Patients often develop features of the anticholinergic toxidrome such as delirium, mydriasis, dry mucous membranes, tachycardia, hyperthermia, mild hypertension, urinary retention, and slow GI motility. CNS toxicity can include lethargy, coma, myoclonic jerks, and seizures. Sinus tachycardia is the most common cardiovascular manifestation of toxicity. Patients can also develop widening of the QRS complex, premature ventricular contractions, and ventricular arrhythmias. Initial management should be directed to supporting vital functions. ECG should be obtained as soon as possible and followed serially to monitor for progression of toxicity. Sodium bicarbonate is the antidote of choice and works via overcoming the sodium channel blockade by providing a sodium load and via inducing an alkalosis to decrease drug binding to sodium channels. Indications for sodium bicarbonate include a QRS duration > 100 ms, ventricular dysrhythmias, and hypotension. Flumazenil (A) is the antidote for benzodiazepine toxicity. N-acetylcysteine (B) is the treatment for acetaminophen poisoning. Naloxone (C) is the antidote for opioid toxicity.

You have been monitoring a 52-year-old perimenopausal woman's hot flashes. She has not had a hysterectomy. Her symptoms have been so mild that she does not require medication. However, for the past two months, her hot flashes have increased in frequency, duration and intensity. She is now asking for a medication. Which of the following is the most appropriate for medical management of her moderate to severe symptoms? Androstenedione alone Androstenedione plus estrogen Estrogen alone Estrogen plus progestin

Correct Answer ( D ) Explanation: The main symptom of a hot flash is a sensation of heat which typically begins in the chest and face which can become generalized. A hot flash may last for a few minutes and occur several times a day, being more prevalent in the evening hours. Hot flashes can affect up to 75% of perimenopausal women. These vasomotor symptoms, which are felt to be due to the effect of declining estrogen on the hypothalamus, are felt to be due to inappropriate peripheral vasodilation which leads to a drop in core body temperature. Associated symptoms include shivering, palpitations and perspiration. Risks factors include tobacco use and obesity. Other than the discomfort of warmth, one of the most distressing side effect of hot flashes is sleep disturbance with resultant fatigue. Most women with mild symptoms do not require medication management. When symptoms become moderate to severe, estrogen therapy is recommended, either oral or transdermal. In women who have not had a hysterectomy, estrogen plus progestin is recommended to prevent endometrial hyperplasia. Androstenedione (A & B) is a precursor hormone in the biosynthesis of estrogen. Its transformation requires an ovary to be receptive to luteinizing hormone, a process which does not occur in menopause. Estrogen alone (C) is the treatment in women who have had a hysterectomy.

A 44-year-old man with a history of intravenous opioid use presents to the Emergency Department with fever, cough, and hemoptysis. Vital signs include BP 110/65 mm Hg, HR 120 beats per minute, RR 20 breaths per minute, and T 103.4°F. On auscultation of the chest, you hear a faint systolic ejection murmur. Which of the following would you expect to see on physical examination? Diffuse erythroderma Palpable purpura Positive Nikolsky sign Splinter hemorrhages

Correct Answer ( D ) Explanation: This patient is presenting with signs and symptoms of infectious endocarditis. Dermatologic and ocular manifestations of endocarditis are important indicators of the diagnosis. Roth spots are retinal hemorrhages with central clearing seen on funduscopic examination. Osler nodes are painful nodules on fingers and toes. Janeway lesions are painless erythematous plaques on the palms and soles. Splinter hemorrhages occur beneath the nails due to septic emboli. Risk factors for infectious endocarditis include rheumatic heart disease, congenital or acquired valvular disease, and intravenous drug use. Left-sided endocarditis involves either the aortic or mitral valve. It is more common than right-sided endocarditis. Organisms often implicated in left-sided endocarditis include Streptococcus viridans, Staphylococcus aureus, and those in the Enterococcus family. Complications include systemic infarcts from septic emboli. Right-sided endocarditis involves either the pulmonic or tricuspid valve. It is classically seen in intravenous drug users. Organisms implicated in right-sided endocarditis include Staphylococcus aureus, Streptococcus pneumoniae, and gram negative bacteria. Presenting symptoms often include fever, cough, hemoptysis, chest pain, and dyspnea. Diagnosis is made by having either both major criteria, 1 major and 3 minor criteria, or 5 minor criteria. Major criteria include 2 positive blood cultures with at least 3 sets sent one hour apart of organisms common to infectious endocarditis or abnormal echocardiography with either visible vegetation, new valvular regurgitation, prosthetic valve dehiscence, or myocardial abscess. Echocardiography is the hallmark of imaging for endocarditis and is preferably done via the transesophageal route. Minor criteria include predisposing risk factors or IV drug use, fever, vascular events such as septic emboli and Janeway lesions, immunologic events such as Osler nodes or Roth spots, echocardiographic findings consistent with endocarditis not meeting major criteria, and positive blood cultures not meeting major criteria. Management includes antibiotics for the suspected organism based on the clinical situation. Diffuse erythroderma (A) is the characteristic rash of toxic shock syndrome. Palpable purpura (B) is seen with Henoch-Schonlein purpura and disseminated intravascular coagulation. Positive Nikolsky sign (C) is characteristic of diseases that cause the top layers of skin to slough off with minimal pressure. This is seen in pemphigus vulgaris, staphylococcal scalded skin syndrome, and toxic epidermal necrolysis.

A 32-year-old woman presents with abdominal pain, nausea, vomiting and change in skin color for 6 days. She states that she had unprotected intercourse 4 weeks ago. Which of the following tests indicates acute infection with hepatitis B as the cause of the patient's symptoms? Antibody to hepatitis B e antigen (Anti-HBe) Antibody to hepatitis B surface antigen (Anti-HBs) IgG antibody to B core antigen (Anti-HBc-IgG) IgM antibody to B core antigen (Anti-HBc-IgM)

Correct Answer ( D ) Explanation: This patient presents with symptoms consistent with acute Hepatitis B virus (HBV) infection. HBV is primarily transmitted through parenteral exposure (needle stick, intravenous drug use) or through unprotected intercourse. Transmission through blood transfusion is rare due to advances in screening techniques. Acute viral hepatitis presents with malaise, fever, anorexia, nausea, vomiting, abdominal discomfort and diarrhea. Often, jaundice leads patients to consult a physician. Fulminant hepatitis is characterized by acute onset of hepatic failure and encephalopathy over a short period of time (usually days). Measurement of hepatic enzymes can demonstrate 10 - 100 fold elevations of aspartate aminotransferase (AST) and alanine aminotransferase (ALT) levels. ALT is usually elevated to a greater degree than AST in viral hepatitis (the reverse is usually true in alcoholic hepatitis). Hyperbilirubinemia can be moderate (5 - 10 mg/dl) or severe (15 - 25 mg/dl) and usually presents days to weeks after the onset of symptoms. Both direct and indirect bilirubin will be elevated in viral hepatitis. Serum testing can also be used to diagnose the type of viral hepatitis. Acute hepatitis B is characterized by the presence of IgM antibody to B core antigen (Anti-HBc-IgM). Anti-HBs (B) cannot be used to diagnose acute HBV infection as it does not differentiate between active, past or immunization to Hepatitis B. IgG antibody to B core antigen (Anti-HBc-IgG) (C) is produced in response to core antigen later in course and persists for life. The presence of antibody to HBV e antigen (Anti-HBe) (A) indicates resolving infection and low infectivity.

A 45-year-old man presents to the clinic with several weeks of dull, gnawing epigastric pain that is usually relieved by eating. He has been taking omeprazole for five weeks, but has had no improvement. His only other daily medication is ibuprofen for knee pain. An abdominal exam is normal and fecal occult blood testing is negative. The next best step in management includes which of the following diagnostic tests? Abdominal CT Barium upper gastrointestinal series H. pylori fecal antigen assay Upper endoscopy

Correct Answer ( D ) Explanation: Upper Endoscopy is the next best step in management of this patient's epigastric pain because it will aid in visualization of a gastric or peptic ulcer and allow for biopsy if necessary. Based on the location and description of this patient's pain, it is likely that he has peptic ulcer disease (PUD). Patients with PUD typically complain of epigastric pain that is aching or "hunger-like" and may be relieved with food or antacids for 2-4 hours. It is not uncommon for patients to be awakened with pain, particularly with duodenal ulcers. This pain may persist on a regular basis for weeks, months, or years; a change in character of pain may indicate a perforation. Patients with gastric ulcers may also complain of nausea and anorexia. However, any significant vomiting or weight loss may indicate a more serious gastric obstruction or malignancy. In uncomplicated PUD, laboratory findings are usually normal; anemia or leukocytosis may occur in the setting of a perforation. An upper endoscopy is the diagnostic procedure of choice. Biopsies of borders should be taken, as a small percentage of ulcers are malignant. Peptic ulcers are typically treatable with acid anti-secretory agents such as proton-pump inhibitors and H2-receptor antagonists. Removal of offending agents, such as NSAIDs, is advisable when possible. Failure to have improvement in 4-8 weeks may indicate an underlying cause of PUD, including H. pylori infection, or excessive serum gastrin levels such as in Zollinger-Ellison syndrome. An abdominal CT (A) is not appropriate because it will not sufficiently allow for visualization of a peptic or gastric ulcer. This choice would be appropriate for a patient with established PUD in whom there may be a PUD-related complication such as perforation or obstruction. A barium gastrointestinal series (B) is less appropriate for this patient because it is not able to visualize ulcers as clearly as an upper endoscopy. An H. pylori antigen assay (C) may be appropriate in this patient once an ulcer is confirmed by upper endoscopy. However, this test should only be used in-patient in whom an ulcer has already been diagnosed.

Capture beats and fusion beats confirm the diagnosis of which cardiac dysrhythmia? Atrial fibrillation Supraventricular tachycardia Ventricular fibrillation Ventricular tachycardia

Correct Answer ( D ) Explanation: Ventricular tachycardia can be diagnosed when capture beats or fusion beats are seen in the setting of a wide complex tachycardia. This is especially helpful when trying to distinguish between ventricular tachycardia and supraventricular tachycardia with aberrant conduction. Supraventricular tachycardia with aberrancy can produce a wide complex tachycardia that mimics ventricular tachycardia. It is important to distinguish between the two because ventricular tachycardia is a more serious condition and treatment is different. During ventricular tachycardia, there is AV dissociation in which independent pacing of the atria and ventricles occurs. The SA node still paces the atria, but the larger ventricular complexes often hide the P waves. However, P waves can be seen occasionally. When a sinus paced depolarization is able to conduct to the ventricles, it can produce a normal-appearing QRS; this is called a capture beat. More commonly, an atrial depolarization is able to conduct only partially before it encounters depolarization coming from the ventricles; this produces a fusion beat, which is a blending of a normal QRS with a PVC like complex. Capture beats and fusion beats do not occur during supraventricular tachycardia, making their presence useful in differentiating the two. Capture and fusion beats are not seen in atrial fibrillation (A), supraventricular tachycardia (B), or ventricular fibrillation (C).

Question: What is the most common bacterial cause of UTI?

E. coli Rapid Review Cystitis F > M E. coli > S. saprophyticus Dysruia, ↑ frequency, urgency, hematuria Pyuria, bacteruira, urine leukocyte esterase + Urine nitrites: gram-negative organisms Acute uncomplicated cystitis: TMP-SMX, nitrofurantoin, or fluoroquinolone for 3-5 days Acute uncomplicated cystitis with comorbid conditions: TMP-SMX, nitrofurantoin, or fluoroquinolone for 7 days

You perform an arthrocentesis on a patient with knee pain. Synovial fluid analysis reveals a WBC 5,000 cells/µL with 70% PMNs. Which of the following is the most likely diagnosis? Hemorrhagic effusion Inflammatory arthritis Noninflammatory arthritis Septic arthritis

Explanation: The analysis of synovial fluid is essential for identifying crystalline and suppurative causes of acute arthritis. The WBC count dictates the inflammatory class of the fluid. However, an analysis is often difficult because there is significant overlap between entities. A very high fluid WBC or PMN pleocytosis indicates infection, but a modest elevation does not exclude it. Therefore, cell counts should be used as a guide rather than definitive diagnosis. WBCs between 2,000 and 75,000 are often categorized as inflammatory. However, a WBC over 50,000 has a likelihood ratio of 7.7 for septic arthritis. Hemorrhagic effusions (A) are caused by trauma to the knee or coagulopathy. The fluid is grossly bloody and may contain fat droplets (lipohemarthrosis). Noninflammatory arthritis (C) such as osteoarthritis will have <3,000 WBCs on arthrocentesis. The likelihood ratio for septic arthritis (D) increases as the joint WBC count rises. A WBC count <25,000 has a likelihood ratio of 0.32, >25,000 has a likelihood ratio of 2.9, >50,000 has a likelihood ratio of 7.7.

A five-year-old girl is brought to the clinic for evaluation of petechiae. The parents deny fever, decreased appetite, bone or joint pain, or weight loss. She did have a cold and runny nose about a month ago. On physical exam, the girl appears well with normal vital signs, petechiae and purpura on the upper and lower extremities. There are no signs of mucosal bleeding with normal lymph nodes, liver, or spleen. Laboratory studies reveal platelet count of 60,000/microL and an otherwise normal complete blood count. Which of the following is the most likely diagnosis? Hemolytic uremic sydrome Immune thrombocytopenia Leukemia Thrombotic thrombocytopenic purpura

Explanation: The girl has symptoms and laboratory findings consistent with immune thrombocytopenia (ITP). ITP is one of the most common causes of symptomatic thrombocytopenia in children. There may be a history of a prior infection within the past month. Other than the prior history of infection, the history generally does not reveal any significant findings. There are no systemic symptoms, nor any prior history of bleeding or significant disease. Almost all patients diagnosed with ITP have signs of cutaneous bleeding that include petechiae, purpura, and ecchymoses. Mucosal bleeding that involves the nasal passages, buccal and gingival surfaces may also be present. Other than mucocutaneous bleeding, patients usually appear well. On physical examination, there is no significant enlargement of lymph nodes, liver, or spleen. Laboratory findings reveal thrombocytopenia that is usually the only abnormality detected. A platelet count of < 100,000/microL is used to define thrombocytopenia in ITP. The white blood cell count and differential, hemoglobin concentration, and other red cell indices are generally normal. Hemolytic uremic syndrome (A) is a microangiopathic disorder that is characterized by hemolytic anemia, thrombocytopenia, and acute renal injury, which may progress to oliguria. Leukemia (C) presents with systemic symptoms like fever, bone and joint pain, or weight loss as well as hepatosplenomegaly, lymphadenopathy, leukocytosis, and significant anemia. Thrombotic thrombocytopenic purpura (D) also presents with thrombocytopenia, but other manifestations include schistocytes on the peripheral smear, anemia, and neurologic abnormalities, which were not present on the patient.


Kaugnay na mga set ng pag-aaral

Maryland Motorcycle Learners Permit Test

View Set

UIL Social Studies 2023-2024: Related Terms

View Set

Proton Pump Inhibitor Therapy Sherpath

View Set

Math: Squares and Square Roots, Lesson 18

View Set

Lesson 22: Chapter 22 Bonding in Ionic Compounds

View Set

Canvas Briefing: Nurturing - Quizlet

View Set

Ch 11 P SC Voting, Campaigns, and Elections

View Set

VNR201 chap 3, VNR201 chap 1, VNR201 chap 2

View Set